Haem1 Flashcards

1
Q

In a patient with a deep venous thrombosis, for which of the following factors occurring in isolation at the time of the initial thrombotic event is there strongest indication for extending anticoagulation beyond six months?

A. Heterozygosity for factor V Leiden mutation.

B. Presence of a lupus inhibitor/anticoagulant.

C. Absence of a provoking stimulus.

D. Presence of an elevated homocysteine level.

E. Heterozygosity for prothrombin G20210A gene mutation.

A

B

How well did you know this?
1
Not at all
2
3
4
5
Perfectly
2
Q

QUESTION 50

Which one of the following statements is the best description of the molecular basis of a chimaeric protein (eg Bcr-Abl)?

A. Formation of disulphide bonds between cytosine residues on different peptides.

B. Post-transcription ligation of non-homologous messenger RNA (mRNA) molecules.

C. Hybrid messenger RNA (mRNA) synthesis from a bi-directional promoter.

D. Deletion of an exon from one gene.

E. In-frame ligation of the 5-end of one gene to the 3-end of another.

A

E

How well did you know this?
1
Not at all
2
3
4
5
Perfectly
3
Q

Give examples of diseases for each RBC-opathy

  1. Abnormality of GPI-linked proteins
  2. Red cell enzyme deficiency
  3. Haemoglobin abnormality.
  4. Red cell membrane abnormality.
  5. Haematinic deficiency. B12/Folate
A
  1. Paroxysmal Nocturnal Haemoglobulinemia.
  2. G6PD, Pyruvate Kinase deficiency
  3. Thalassemia, Sickle cell anaemia
  4. Spherocytosis, elliptocytosis
  5. B12/Folate/Iron deficiency
How well did you know this?
1
Not at all
2
3
4
5
Perfectly
4
Q

Why do you get leg ulcers in haemolytic disorders?

A

Leg ulcers may develop in patients with sickle cell anemia and other hemolytic disorders, as a result of decreased red blood cell (RBC) deformability and endothelial changes

How well did you know this?
1
Not at all
2
3
4
5
Perfectly
5
Q

QUESTION 51

Which of the following best explains the morphological appearance of red cells pictured below?

A. Abnormality of GPI-linked proteins.

B. Red cell enzyme deficiency.

C. Haemoglobin abnormality.

D. Red cell membrane abnormality.

E. Haematinic deficiency.

A

D. This is elliptocytosis

How well did you know this?
1
Not at all
2
3
4
5
Perfectly
6
Q

Of the following antiplatelet actions, which best describes the mechanism of clopidogrel?

A. Phosphodiesterase inhibition.

B. COX1 acetylation.

C. ADP receptor blocking. ADPase on endothelial cells. Granules released from platelets which leads to further platelet aggregation

D. GP IIb/IIIa blockade.

E. Factor Xa inhibition.

A

C

How well did you know this?
1
Not at all
2
3
4
5
Perfectly
7
Q

QUESTION 64

Which of the following is most characteristic of a patient with haemophilia A with 8% baseline factor VIII activity?

A. Spontaneous cutaneous purpura.

B. Spontaneous gastrointestinal haemorrhage.

C. Spontaneous deep muscle bleeds.

D. Spontaneous haemarthroses.

E. No spontaneous bleeding.

A

E. >5% activity = no bleeding

A is platelets not factors.

B doesn’t happen. (Can get haematoma)

How well did you know this?
1
Not at all
2
3
4
5
Perfectly
8
Q

A 69-year-old woman presents with severe pancytopenia and peripheral blood macrocytosis associated with a low serum vitamin B12 level.

Which one of the following investigation results most strongly supports a diagnosis of pernicious anaemia?

A. Positive anti-parietal cell antibody.

B. Positive intrinsic factor antibody.

C. Elevated fasting plasma homocysteine level.

D. Elevated fasting serum gastrin level.

E. Reduced red cell folate level.

A

B

How well did you know this?
1
Not at all
2
3
4
5
Perfectly
9
Q

Which of the following findings is most specific for the diagnosis of pernicious anaemia?

A. Increased serum homocysteine.

B. Increased serum methylmalonate.

C. Positive gastric parietal cell antibodies.

D. Increased serum gastrin.

E. Positive intrinsic factor antibodies.

A

E. Highly specific

How well did you know this?
1
Not at all
2
3
4
5
Perfectly
10
Q

A 21-year-old Filipino woman has been referred for investigation of anaemia. Her blood counts and iron studies are as follows:

Which of the following is the most likely diagnosis?

A. alpha+-thalassaemia trait (a-/aa).

B. alpha0-thalassaemia trait (–/aa).

C. HbH disease (–/-a).

D. beta0-thalassaemia trait.

E. beta+-thalassaemia trait.

A

C. HbH disease (–/-a).

A & B only give mild anaemia

In D&E, HbF makes up the majority of Hb with variable amounts of HbA2

How well did you know this?
1
Not at all
2
3
4
5
Perfectly
11
Q

A 65-year-old woman, established on haemodialysis for nine months, is maintained on erythropoietin 4,000 units twice a week intravenously. Previously her haemoglobin (Hb) had remained steady in the range of 105 – 115 gm/L. She now presents with a Hb of 89 gm/L on routine testing. There have been no changes to her erythropoietin dose or other medications. Her mean corpuscular volume (MCV) is 84 fL [78-98], her serum iron is 13 μmol/l [13-35], iron saturation is 18% [15-46], serum ferritin is 630 μg/l [20-300] and her reticulocyte count is 30 x 109/L [12-100].

Which of the following is the most likely cause for her erythropoietin resistance?

A. Inflammation.

B. Pure red cell aplasia.

C. Vitamin B12 deficiency.

D. Iron deficiency.

E. Hyperparathyroidism.

A

A

How well did you know this?
1
Not at all
2
3
4
5
Perfectly
12
Q

A 78-year-old woman has a history of hypertension on amlodipine, and has been found to have a persistently elevated platelet count for several months. She presents to the emergency department with abdominal pain. Her full blood examination and serum biochemistry are as follows:

Which of the following is the most likely cause of her elevated serum potassium?

A. Pseudohyperkalaemia.

B. Renal tubular acidosis.

C. Chronic renal impairment.

D. Haemolytic anaemia.

E. Lactic acidosis.

A

A

How well did you know this?
1
Not at all
2
3
4
5
Perfectly
13
Q

A previously well 37-year-old man presents with lethargy and easy bruising over several weeks. Physical examination is unremarkable apart from occasional bruises.

His full blood examination shows:

His blood film is shown below.

The most likely diagnosis is:

A. essential thrombocythaemia.

B. occult haemorrhage.

C. primary myelofibrosis.

D. occult carcinoma.

E. chronic myeloid leukaemia.

A

A

How well did you know this?
1
Not at all
2
3
4
5
Perfectly
14
Q

A 47-year-old woman has coagulation studies performed as part of preoperative assessment prior to elective hysterectomy for menorrhagia.

Which of the following is the most likely interpretation of these results?

A. Tranexamic acid therapy.

B. Disseminated intravascular coagulation.

C. Heparin exposure.

D. Von Willebrand disease.

E. Lupus anticoagulant.

A

E

How well did you know this?
1
Not at all
2
3
4
5
Perfectly
15
Q

Which of the following is the most common mechanism of a Febrile Non-Haemolytic Transfusion reaction (FNHTR) to transfusion of packed red cells?

A. Bacterial contamination of donor product.

B. Immediate hypersensitivity to proteins of donor origin.

C. Interaction between donor antibodies with recipient granulocytes.

D. Interaction between donor granulocytes with recipient antibodies.

E. Interaction between donor red cells with recipient red cell antibodies.

A

D

How well did you know this?
1
Not at all
2
3
4
5
Perfectly
16
Q

QUESTION 29

Which of the following is the greatest cause of mortality occurring more than 100 days following allogeneic peripheral blood cell stem transplantation for acute leukaemia?

A. Hepatic veno-occlusive disease.

B. Graft-versus-host disease.

C. Transplantation-related lung injury.

D. Transplantation-related infection.

E. Recurrent leukaemia.

A

E

How well did you know this?
1
Not at all
2
3
4
5
Perfectly
17
Q

A 72-year-old woman who is taking warfarin for an embolic stroke secondary to atrial fibrillation presents to the emergency department after a simple fall. On examination she has a few small bruises on her legs but is otherwise well. Her pulse is 68 bpm and irregular and her blood pressure is 175/95 mmHg. Her full blood examination is normal however her serum creatinine is elevated at 170 mcg/L [50-100 mcg/L] and her INR is elevated at 7.1.

In addition to regular monitoring, which of the following is the best management strategy for her elevated international normalised ratio (INR)?

A. Withhold warfarin.

B. Withhold warfarin and administer Vitamin K1.

C. Withhold warfarin and administer Vitamin K1 and prothrombin complex concentrate.

D. Withhold warfarin and administer Vitamin K1, prothrombin complex concentrate with fresh plasma.

E. Withhold warfarin and administer prothrombin complex concentrate with fresh plasma.

A

B

How well did you know this?
1
Not at all
2
3
4
5
Perfectly
18
Q

A 58-year-old woman has been unwell for several weeks with back pains. She has no past medical history of note. She is a smoker of 25 pack years. On examination she is tender over the mid thoracic spine and over several ribs. The rest of the examination is unremarkable. Blood tests show:

A CXR shows lytic lesions in several thoracic vertebrae with clear lung fields. Spinal X-rays confirm several lytic vertebral lesions. Immunoglobulins are normal. Serum and urine electrophoresis and immunoelectrophoresis are negative. Mammography is normal. CT scanning of chest and abdomen shows the vertebral lesions but no other masses, no organomegaly and no lymphadenopathy. Thyroid and pelvic ultrasound are normal. A bone scan shows no increased uptake.

Which of the following tests should next be performed?

A. 24 hour urine calcium.

B. Bone marrow examination.

C. Biopsy of a bone lesion.

D. Vitamin D levels.

E. Parathyroid hormone related peptide levels.

A

B. This woman presents with the CRAB acronym of multiple myeloma - and it’s likely nonsecretory. The other option would be serum free light chains which picks up nonsecretory disease

How well did you know this?
1
Not at all
2
3
4
5
Perfectly
19
Q

A 26-year-old female has been found to be homozygous for the HFE gene C282Y polymorphism, when screened for haemochromatosis. Her serum ferritin is 18 micrograms/L [15-200 micrograms/L] and her transferrin saturation is 82% [

Which of the following is the most appropriate next step in her management?

A. Venesection.

B. Observation.

C. Liver biopsy.

D. Imaging of liver for iron loading.

E. Desferrioxamine.

A

B

How well did you know this?
1
Not at all
2
3
4
5
Perfectly
20
Q

Question 51

The following mutations (Cys282Tyr and His63Asp) are associated with hereditary haemochromatosis. Which one of the following genotypes provides the greatest risk for the development of clinical disease?

A. Heterozygous Cys282Tyr.

B. Heterozygous His63Asp.

C. Double-heterozygote for Cys282Tyr and His63Asp.

D. Homozygous Cys282Tyr.

E. Homozygous His63Asp.

A

D

How well did you know this?
1
Not at all
2
3
4
5
Perfectly
21
Q

QUESTION 78

Which of the following situations is most likely to result in a false negative FDG-PET (Fluorodeoxyglucose-Positron Emission Tomography) scan when restaging a patient with aggressive lymphoma post-chemotherapy?

A. Necrosis at site of previous involvement.

B. Fibrosis at site of previous involvement.

C. Brown fat.

D. Uncontrolled diabetes.

E. Recent surgery.

A

D

How well did you know this?
1
Not at all
2
3
4
5
Perfectly
22
Q

A 67-year-old man with longstanding rheumatoid arthritis treated with methotrexate, prednisolone and celecoxib presents unwell with fatigue and anorexia. On examination his arthritis is quiescent but he has two mouth ulcers. His full blood examination is shown below.

The most appropriate drug to administer is:

A. folic acid.

B. filgrastim.

C. folinic acid.

D. cholestyramine.

E. prednisolone.

A

C

How well did you know this?
1
Not at all
2
3
4
5
Perfectly
23
Q

Which of the following transfusion reactions is leucodepletion most effective in preventing?

A. Febrile non-haemolytic transfusion reaction.

B. Transfusion associated graft versus host disease.

C. Anaphylaxis.

D. Transfusion related acute lung injury.

E. Delayed haemolytic transfusion reaction.

A

A

How well did you know this?
1
Not at all
2
3
4
5
Perfectly
24
Q

QUESTION 7

Hepcidin and ferroportin regulate systemic iron absorption by inhibition of:

A. enterocyte brush border iron reduction.

B. enterocyte apical iron uptake.

C. iron oxidation by hephaestin.

D. enterocyte basolateral iron release.

E. iron binding to transferrin.

A

D

How well did you know this?
1
Not at all
2
3
4
5
Perfectly
25
Q

QUESTION 38

Other than bacterial infections, opportunistic infection with which of the following is most likely to occur in a patient after induction chemotherapy for acute myeloid leukaemia (AML)?

A. Candida albicans.

B. Mycobacterium tuberculosis.

C. Cytomegalovirus.

D. Cryptosporidium.

E. Toxoplasma gondii.

A

A

How well did you know this?
1
Not at all
2
3
4
5
Perfectly
26
Q

A blood film showing acanthocytes and Howell-Jolly bodies is taken from a patient with

coeliac disease.

Which of the following is the most likely explanation for this appearance?

A. Malnutrition.

B. Hyposplenism.

C. Iron deficiency.

D. Hypothyroidism.

E. Folate deficiency.

A

B

How well did you know this?
1
Not at all
2
3
4
5
Perfectly
27
Q

QUESTION 46

Some autosomal recessive diseases have high prevalence in particular populations, even though they

are often fatal (e.g. Alpha thalassaemia in South East Asia; G6PD deficiency in the Mediterranean).

Which of the following is the most likely explanation?

A. Consanguinity.

B. High mutation rates in specific populations.

C. Survival advantage in heterozygous carriers.

D. Survival advantage in wild type homozygotes.

E. Founder effect.

A

C

How well did you know this?
1
Not at all
2
3
4
5
Perfectly
28
Q

A 30-year-old female presents with a painful swollen left calf. Duplex study demonstrates a proximal

deep vein thrombus extending into the ileofemoral venous system. The following laboratory results

are obtained:

Prothrombin time (PT) 16 sec [11 – 18 sec]

Activated partial thromboplastin time (APTT) 43 sec [25 – 36 sec]

Dilute Russell Viper venom test Lupus anticoagulant detected

Anticardiolipin IgG negative

Anticardiolipin IgM negative

Anti-beta 2 glycoprotein 1 moderate positive

Treatment with anticoagulation with heparin followed by warfarin therapy is commenced. Repeat

testing at 12-weeks confirms a positive anti-beta 2 glycoprotein 1. What is the most appropriate

duration of anticoagulation?

A. 3 months.

B. 6 months.

C. 12 months.

D. 24 months.

E. Lifelong.

A

E

How well did you know this?
1
Not at all
2
3
4
5
Perfectly
29
Q

QUESTION 64

A 37-year-old male is found to have some persistently enlarged lymph nodes (<2 cm diameter) in the

left posterior triangle. A biopsy reveals follicular, Grade 1, B-cell non-Hodgkin lymphoma. Staging

procedures including computed tomography (CT) scan, positron emission tomography (PET) scan and bone marrow biopsy confirm the disease is confined to a single lymph node region in the neck.

Which of the following management strategies is most appropriate?

A. Watch and wait.

B. Oral chlorambucil.

C. Radiotherapy.

D. Multiagent chemotherapy.

E. Combined modality therapy.

A

C

How well did you know this?
1
Not at all
2
3
4
5
Perfectly
30
Q

QUESTION 68

A 29-year-old woman is referred for evaluation after she was found to be heterozygous for

factor V Leiden mutation on screening. This screening was initiated when her great aunt sustained a

calf vein thrombosis following hip arthroplasty. She is currently 16-weeks pregnant and has no

personal history of venous thromboembolic disease.

Which of the following should be recommended for the remainder of her pregnancy?

A. Low molecular weight heparin.

B. Aspirin.

C. No thromboprophylaxis.

D. Surveillance ultrasonography of the lower limbs.

E. Warfarin.

A

C

How well did you know this?
1
Not at all
2
3
4
5
Perfectly
31
Q

QUESTION 74

A 67-year-old diabetic man with atrial fibrillation is taking warfarin for primary prophylaxis and is

scheduled for inguinal hernia repair. His echocardiogram shows no structural abnormality. His

international normalised ratio (INR) is 2.9. Which of the following preoperative anticoagulant

management strategies is most appropriate?

A. Withhold warfarin therapy for four days before surgery and administer low-molecular-weight

heparin (LMWH) for these four days, ceasing 24 hours prior to surgery.

B. Withhold warfarin therapy four days before surgery and administer vitamin K1 on the eveningprior to surgery if his INR> 1.5.

C. Withhold warfarin for two days prior to surgery.

D. Continue warfarin up until the time of surgery then administer prothrombin complex

concentrate on the day of surgery.

E. Withhold warfarin therapy for four days before surgery. Commence intravenous infusion of

unfractionated heparin two days prior to surgery, ceasing four hours prior to surgery.

A

B

How well did you know this?
1
Not at all
2
3
4
5
Perfectly
32
Q

QUESTION 40

Poisoning with which of the following causes a shift in the Hb-oxygen dissociation curve to the left?

A. Carbon monoxide.

B. Cyanide.

C. Gamma-hydroxy-butyrate (GHB).

D. Methanol.

E. Aspirin.

A

A

How well did you know this?
1
Not at all
2
3
4
5
Perfectly
33
Q

A 74-year-old ex-smoking male with a history of ischaemic heart disease presents with a hip fracture requiring surgical fixation within a few days. He is plethoric and there are no specific respiratory or cardiac abnormalities on examination. Chest X-ray is normal and ECG shows left bundle branch block.

His full blood count shows:

Which one of the following would be the most appropriate initial therapy?

A. Venesection.

B. Intravenous radioactive phosphorus.

C. Hydroxyurea.

D. Therapeutic anticoagulation.

E. Interferon α.

A

A. It is critical that the thrombocytosis and increased RBC mass is controlled pre-op otherwise the risk of arterial thrombosis is unacceptably high.

How well did you know this?
1
Not at all
2
3
4
5
Perfectly
34
Q

A 48-year-old previously well man, currently smoking 10 to 20 cigarettes/day, presents with the progressive onset over several months of lethargy and headache. He takes no medications.

Examination reveals him to be plethoric and hypertensive with no other significant findings.

Full blood examination shows:

Which one of the following would most strongly support a diagnosis of polycythaemia vera?

A. Raised serum vitamin B12 level.

B. Elevated total red cell mass.

C. Normal marrow karyotype.

D. Elevated serum uric acid level.

E. Lowered serum erythropoietin level.

A

Answer: E

How well did you know this?
1
Not at all
2
3
4
5
Perfectly
35
Q

A 72-year-old man with a history of chronic obstructive pulmonary disease (COPD) presents with an intercurrent lower respiratory tract infection. Apart from plethora and pulmonary signs associated with his respiratory disease, no other abnormalities are found.

Full blood examination shows:

The most likely cause of the elevated haematocrit is:

A. dehydration.

B. chronic hypoxaemia.

C. polycythaemia vera.

D. spurious polycythaemia.

E. low affinity haemoglobin.

A

B. The EPO level is raised.

How well did you know this?
1
Not at all
2
3
4
5
Perfectly
36
Q

What are the main features of polycythaemia vera?

A
  • JAK2 mutation
  • increased RBC mass
  • Splenomegaly
  • Normal arterial O2
  • elevated WBC and platelets
How well did you know this?
1
Not at all
2
3
4
5
Perfectly
37
Q

What is the mechanism in polycythaemia vera?

A

Mechanism

Acquired JAK2 exon mutation (95% of cases) —> proliferation of all three haematopoietic cells lines (megakaryocytes, RBCs, myeloid) —> prominent overproduction of RBCs and to a lesser extent, WBCs and platelets w/ low EPO level.

How well did you know this?
1
Not at all
2
3
4
5
Perfectly
38
Q

What are the clinical findings in polycythaemia vera?

A

Clinical findings

Expanded blood volume & viscosity —> headache, dizziness, blurred vision, fatigue

Histamine release from basophilia —> fatigue

Epistaxis and GI bleeds —> abnormal platelet function and mucosal blood vessel engorgements

Plethora, engorged retinal veins

Splenomegaly

Thrombosis is major cause of morbidity and death - due to viscosity and abnormal platelets. MUST control disease before any surgery

How well did you know this?
1
Not at all
2
3
4
5
Perfectly
39
Q

Epidemiology of polycythaemia vera?

A

Epidemiology

Men in their 60s, v rare under 40

How well did you know this?
1
Not at all
2
3
4
5
Perfectly
40
Q

What are the lab findings in polycythaemia vera?

A

Elevated HCT - >54/51% M/F respectively

Elevated RBC mass

Elevated WBC and PLT counts

Normal morphology of all three lines

Basophilia and eosinophilia

Low EPO levels

JAK2 mutation positive

BMBx - hyper cellular, panhyperplasia w/ absent iron stores (transferred to circulating RBCs)

Very high B12 due to secretion of transcobalamin III from WBCs

Hyperuricaemia

Iron deficiency from bleeding & phlebotomy - microcytosis w/ normal HCT

Elliptocytosis from progressive hypersplenism

How well did you know this?
1
Not at all
2
3
4
5
Perfectly
41
Q

What is the diagnostic criteria for polycythaemia vera?

A

Major Criteria (2 Mj + 1Mn or 1st Mj + 2Mn)

Hb more than 185 in men, 165 g in women, or elevated red cell mass greater than 25%

Presence of JAK2 617V greater than F or other similar mutations

Minor Criteria

BMBx: hypercellularity w/ prominent erythroid, granulocytic, and megakaryocytic proliferation.

Serum erythropoietin level below normal range.

Endogenous erythroid colony formation in vitro.

How well did you know this?
1
Not at all
2
3
4
5
Perfectly
42
Q

What is the differential diagnosis of polycythaemia vera?

A

Spurious - elevated HCT due to volume contraction / diuretis

Secondary

  • Hypoxia and smoking - carboxyhaemoglobin levels may be elevated
  • renal lesions (tumor, EPO secreting tumor (rare))
  • Abnormal Hb (rare)
  • CML if very high WCC
  • Myelofibrosis if abnormal RBC morphology and nucleated RBCs
  • ET is ultra high PLT count
How well did you know this?
1
Not at all
2
3
4
5
Perfectly
43
Q

What is the treatment of polycythaemia vera?

A

Phlebotomy to maintain HCT <45%, need should gradually decrease as iron stores diminish, avoid supplementation.

Decreases thrombotic complications

Low iron diet increases intervals between phlebotomy

If unable (high req, thrombocytosis, intractable pruritus) —> hydroxurea 1st, anagrelide 2nd

Reduction of platelet count reduces complications

Avoid alkylating agents - increases risk of conversion to acute leukaemia

All patients should receive aspirin, reduces thrombosis w/o risk of excessive bleeding

Allopurinol for hyperuricaemia

Antihistamines for itch, SSRIs if intractable

How well did you know this?
1
Not at all
2
3
4
5
Perfectly
44
Q

What is the prognosis of polycythaemia vera?

A

Indolent, median survival 11-15 years

Arterial thrombosis major cause of morbidity and mortality

May convert to myelofibrosis or CML

5% to AML (rare)

How well did you know this?
1
Not at all
2
3
4
5
Perfectly
45
Q

What is pseudohyperkalemia?

A

Pseudohyperkalemia is a rise in the amount of potassium that occurs due to excessive leakage of potassium from cells, during or after blood is drawn.

Usually a lab artefact due to haemolysis during venepecture (excessive vacuum, too fine gauge), excessive tourniquet time, fist clench, delay in processing or tube dropping.

Also can occur in thrombocytosis (>500), leukocytosis (>70), or erythrocytosis (HCT >55%)

How well did you know this?
1
Not at all
2
3
4
5
Perfectly
46
Q

What are the features of essential thrombocythaemia?

A

Features

  • elevated platelet count
  • normal RBC mass
  • absence of BCR-ABL gene
  • uncommon
How well did you know this?
1
Not at all
2
3
4
5
Perfectly
47
Q

What is the mechanism of essential thrombocythaemia?

A

Mechanism

Unknown cause (high freq of JAK2 mutation) —> Proliferation of megakaryocytes —> thrombocytosis —> venous thrombosis

How well did you know this?
1
Not at all
2
3
4
5
Perfectly
48
Q

What is the epidemiology of essential thrombocythaemia?

A

Epidemiology

50-60 year olds, slightly more in women, rare

How well did you know this?
1
Not at all
2
3
4
5
Perfectly
49
Q

What are the clinical findings in essential thrombocythaemia?

A

Clinical findings

Discovery of elevated platelet count

Venous thromboses at unusual sights (i.e mesenteric, hepatic or portal vein

Erythromelalgia (painful burning of the hands)

Mucosal bleeding due to faulty platelets

Splenomegaly in at least 25%

How well did you know this?
1
Not at all
2
3
4
5
Perfectly
50
Q

What are the lab findings in essential thrombocythaemia?

A

Elevated PLT over 1-2000

Mildly elevated WBC w/ immature myeloid forms

Normal haematocrit

Smear: Large platelets, but not giant or degranulated. Normal RBC morphology

BMBx: Megakaryocytosis w/ no other morphologic abnormalities

Cytogenetics: Absent BCR-ABL (need to do to rule out CML)

JAK2 mutation in 50% of cases

How well did you know this?
1
Not at all
2
3
4
5
Perfectly
51
Q

What is the differential diagnosis of essential thrombocythaemia?

A

Reactive thrombocytosis —> PLT count not as high (less than 1000)

Inflammatory disorders (RA, UC, chronic infection)

Iron deficiency —> but thrombocytosis only if significant anaemia

Splenectomy → temporary thrombocytosis

PCV - no erythryocytosis

Myelofibrosis - normal RBC morphology, no nucleated RBCS, no giant degranulated platelets

CML - BCR ABL negative.

How well did you know this?
1
Not at all
2
3
4
5
Perfectly
52
Q

What is the treatment of essential thrombocythaemia?

A

Control platelet count w/ oral hydroxyurea, prevents thrombotic events w/o toxicity

2nd line anagrelide (low dose) - high dose = headache peripheral oedema and CHF

Vasomotor sx (erythromelalgia and parasthesias) —> aspirin

Low dose aspirin may reduce risk of thrombotic complications

Severe bleeding —> plateletpheresis

How well did you know this?
1
Not at all
2
3
4
5
Perfectly
53
Q

What is the prognosis in essential thrombocythaemia?

A

Prognosis

Indolent, high long term survival with control of thrombosis

Late: fibrotic BM, massive splenomegaly and infarction

10-15% risk of progression to myelofibrosis, 1-5% risk of transformation to acute leukaemia

How well did you know this?
1
Not at all
2
3
4
5
Perfectly
54
Q

A 62-year-old man with a history of controlled hypertension presents with worsening lethargy over several months and more recently, weight loss, night sweats and easy bruising. Examination reveals anaemia, bruising and firm splenomegaly 12 cm below the left costal margin; no lymphadenopathy is evident.

Full blood examination reveals:

The blood film shows:

  • teardrop poikilocytosis
  • large degranulated platelet
  • thrombocytopaenia
  • nucleated RBC

The most likely diagnosis is:

A. metastatic carcinoma.

B. megaloblastosis.

C. primary myelofibrosis.

D. systemic lupus erythematosus.

E. chronic myeloid leukaemia.

A

C

How well did you know this?
1
Not at all
2
3
4
5
Perfectly
55
Q

What are the main features of myelofibrosis?

A

Features

  • striking splenomegaly
  • teardrop poikilocytosis
  • leukoerythroblastic bloods w/ giant abnormal plt
  • Hypercellular BM w/ reitculin or collagen fibrosis
How well did you know this?
1
Not at all
2
3
4
5
Perfectly
56
Q

What is the mechanism of myelofibrosis?

A

Mechanism

Increased secretion of platelet-derived growth factor —> bone marrow fibrosis —> extramedullary haematopoiesis (liver, spleen, LNs), reactivation of feral mesenchymal stem cells —> splenomegaly, leukoerythroblastic blood film w/ teardrop poikilocytosis —> anaemia, splenomegaly, BM failure

How well did you know this?
1
Not at all
2
3
4
5
Perfectly
57
Q

What is the epidemiology of myelofibrosis?

A

Epidemiology

>50, insidious

How well did you know this?
1
Not at all
2
3
4
5
Perfectly
58
Q

What are the clinical findings in myelofibrosis?

A

Fatigue due to anaemia

Abdominal fullness and early satiety due to splenomegaly

Massive splenomegaly

Hepatomegaly in >50% of cases

Late: progressive thrombocytopaenia due to BM failure

Painful episodes of splenic infarction

Severe bone pain - esp upper legs

Cachexia

Portal HTN w/ ascites and varices due to hepatic haematopoeisis

Occasionally: transverse myelitis due to epidural myelopoeises

How well did you know this?
1
Not at all
2
3
4
5
Perfectly
59
Q

What are the lab results in myelofibrosis?

A

Labs

Anaemia

Variable WCC (low/N/high)

Smear: Significant poikilocytosis and teardrop RBCs, nucleated RBCs, left shifted myeloid series. Bizarre giant degranulated platelets

Triad: teardrop poikilocytos, leukoerythroblastic blood, giant abnormal platelets

BMBx: Dry tap usually but if early: hyper cellular with reticulin fibres on silver stain demonstrating fibrosis. Later: Collagen, no precursors

JAK2 mutation in 65% of cases, MPL in 40%

How well did you know this?
1
Not at all
2
3
4
5
Perfectly
60
Q

What is the classic finding on blood smear in myelofibrosis?

A

Triad: teardrop poikilocytos, leukoerythroblastic blood, giant abnormal platelets

How well did you know this?
1
Not at all
2
3
4
5
Perfectly
61
Q

What is the ddx of myelofibrosis?

A

Leukoertyhroblastosis —> severe infection, inflammation or infiltrated BM BUT no poikilocytosis/weird platelets

BM fibrosis —> metastatic ca, Hodgkin lymphoma, hairy cell leukaemia, need morphology

CML —> but normal RBCs w/ marked leucocytosis

PCV - elevated haematocrit

ET —> platelet elevation dominant

How well did you know this?
1
Not at all
2
3
4
5
Perfectly
62
Q

What is the treatment of myelofibrosis?

A

Tx

Mild —> nothing

Anaemia —> transfusion +/- EPO

Splenic enlargement / pain / severe thrombocytopaenia /high transfusion requirement —> splenectomy

Bone pain / Pulm htn —> RTx

Curative: Allo SCT

Hydroxurea or lenalidomide may help control anaemia

JAK2 inhibitors not on market yet

How well did you know this?
1
Not at all
2
3
4
5
Perfectly
63
Q

What is the prognosis in myelofibrosis?

A

Prognosis

Median survival - 5 years

End stage—> liver failure, bleeding from thrombocytopaenia, sometimes AML.

How well did you know this?
1
Not at all
2
3
4
5
Perfectly
64
Q

Name the microcytic anaemias

A

Iron deficiency

Thalassemia

Anemia of chronic disease

Lead toxicity

How well did you know this?
1
Not at all
2
3
4
5
Perfectly
65
Q

Name the macrocytic megaloblastic anaemias

A

Vitamin B12 deficiency

Folate deficiency

DNA synthesis inhibitors

How well did you know this?
1
Not at all
2
3
4
5
Perfectly
66
Q

Name the Macrocytic Nonmegaloblastic anaemias

A

Myelodysplasia

Liver disease Reticulocytosis Hypothyroidism

Bone marrow failure state

(eg, aplastic anemia, marrow infiltrative disorder, etc.)

How well did you know this?
1
Not at all
2
3
4
5
Perfectly
67
Q

Name the normocytic anaemias

A

Kidney disease

Mild form of most acquired etiologies of anemia

How well did you know this?
1
Not at all
2
3
4
5
Perfectly
68
Q

Under what circumstances does reticulocytosis occur?

A
  1. acute blood loss,
  2. recent replacement of a missing erythropoietic nutrient
  3. reduced red blood cell survival (ie, hemolysis)
How well did you know this?
1
Not at all
2
3
4
5
Perfectly
69
Q

What causes a severely microcytic anaemia?

Macrocytic?

A

A severely microcytic anemia (mean corpuscular volume [MCV <70 fL) is due either to iron deficiency or thalassemia, while a severely macrocytic anemia (MCV >125 fL) is almost always due to either megaloblastic anemia or to cold agglutinins in blood analyzed at room temperature.

How well did you know this?
1
Not at all
2
3
4
5
Perfectly
70
Q

How and where is dietary iron absorbed and stored?

Why does free iron not circulate?

A
  • RBCs, ferritin and hemosiderin are major storage pools
  • 10% of dietary iron is absorbed
  • Absorbed in stomach, duodenum, jejunum under acidic conditions
  • 70% of iron is in RBCs
  • minor daily losses from sloughing of epithelial cells or insignificant blood losses
  • free iron can produce ROS —> tissue damage, promotion of bacteria (who need it for growth
How well did you know this?
1
Not at all
2
3
4
5
Perfectly
71
Q

Describe the mechanism of iron transport and what affects it.

A

Iron transport

  • across intestinal lumen by ferroportin (it is exported from the basolateral surface of the enterocyte to the circulation)
  • ferroportin also facilitates transport of iron in macrophages and then to apotransferrin (transferrin) in RBCs for haemoglobin synthesis
  • Hepcidin (produced during iron loading and inflammation) promotes degradation of ferroportin by binding it —> decreased release into circulation, sequestration of iron into macrophages, and decreased circulating iron
  • menstrual blood loss plays a major role in iron metabolism

Anaemia of chronic disease —> body iron stores not deficient but not available for RBC production due to hepcidin stimulation

Hepcidin suppression: Fe deficiency, hypoxia, haemorrhagic or haemolytic anaemias —> release of stored iron and increased dietary absorption

In iron overload —> hepcidin deficiency

How well did you know this?
1
Not at all
2
3
4
5
Perfectly
72
Q

What is ferritin and what is it’s role in assessing anaemia?

A

Ferritin

  • intracellular storage protein
  • <30 indicates iron deficiency
  • good marker of total body iron stores
  • low ferritin seen almost only in iron deficiency
  • do not reflect iron stores alone, normal-high levels does not exclude
  • elevated in inflammation, infection, malignancy (blood and solid), liver, kidney disease, high BMI, post-menopause
  • serum iron is poor measure, is a negative acute phase reactant in acute inflammation
  • level testing not currently available
How well did you know this?
1
Not at all
2
3
4
5
Perfectly
73
Q

What is transferrin and what is it’s role in assessing anaemia?

A

Transferrin

  • carrier protein for iron
  • carries iron to transferrin receptor which endocytoses complex into cell which then releases iron for use or stores it
  • liver synthesis apotransferrin so high levels in Fe+ defiency or high oestrogen states, low levels may be due to liver disease or inflame, it is also a negative acute phase reactant
How well did you know this?
1
Not at all
2
3
4
5
Perfectly
74
Q

What is the role of the soluble transferrin receptor in assessing anaemia?

A

Soluble transferrin receptor

  • useful if presence of inflammation is obfuscating result (i.e. in inflammation)
  • upregulated when iron needs are increased (i.e. deficiency, or PCV)
  • not an acute phase reactant
  • testing not currently rebateable
How well did you know this?
1
Not at all
2
3
4
5
Perfectly
75
Q

What is the transferrin saturation? When is it useful?

A

Transferrin saturation

  • calculated ratio between serum iron and TIBC - hence influenced by anything that affects serum iron
  • elevation may be earliest indicator of hereditary haemochromatosis
How well did you know this?
1
Not at all
2
3
4
5
Perfectly
76
Q

What are the major causes of iron deficiency?

A

Chronic blood loss (most important)

  • Esp GI
  • Aspirin/anti-inflams even w/o structural lesion
  • menorrhagia or uterine bleeding
  • repeated blood donations
  • chronic haemoglobinuria (uncommon): traumatic haemolysis (valve), or other intravascular haemolysis i.e. PNH.

Menorrhagia and Pregnancy

  • iron deficient unless supplemented, diet alone doesn’t cut it

Decreased absorption

  • coeliac disease, surgical stomach resection or jejunal bypass
How well did you know this?
1
Not at all
2
3
4
5
Perfectly
77
Q

What are the clinical findings of iron deficiency anaemia?

A

Clinical findings

Anaemia —> fatigue, tachycardia, palpitations, SOBOE

Severe: skin and mucosal changes, smooth tongue, brittle nails, spooning (koilonychia) and cheilosis (inflamed corners of mouth)

Dysphagia due to oesophageal webs (Plummer-Vinson syndrome) in severe iron deficiency

Pica: often for foods not rich in iron

How well did you know this?
1
Not at all
2
3
4
5
Perfectly
78
Q

What are the lab findings in iron deficiency anaemia?

A

Lab findings

Initially iron deficiency w/o anaemia followed by anaemia and normal MCV, then hypo chromic microcytic anaemia

Ferritin <30 (or <15 is v specific but not sensitive)

Ferritin is an acute phase reactant so unreliable in response to inflammation - not ruled out if normal or elevated

Transferrin elevates to compensate leading to transferrin saturations of less than <15% (Serum iron : TIBC) however this ratio will be low in anaemia of inflammation too

Anisocytosis (size) & poikilocytosis (shape) in more severe disease, followed by severe hypochromia, target cells, or pencil/cigar-shaped cells.

How well did you know this?
1
Not at all
2
3
4
5
Perfectly
79
Q

What is the differential diagnosis of iron deficiency anaemia? How do you exclude them?

A

DDx

Anaemia of chronic disease (normal or increased iron stores in BM macrophages, normal or elevated ferritin, serum iron and transferrin saturation is low, TIBC and transferrin n/low

Thalassaemiea - greater degree of microcytosis for level of anaemia with normal or elevated RBC count (unlike every other cause of anaemia). Similar smear to severe iron deficiency

How well did you know this?
1
Not at all
2
3
4
5
Perfectly
80
Q

What is the treatment of iron deficiency anaemia?

A

Treatment

Identify cause

Ferrous sulcate, go slower and lower for compliance

Should be halfway toward normal within 3 weeks, full return to baseline after 2 months, continue for 3-6 months to replenish iron stores

Failure mostly due to non compliance, sometimes poor absorption due to stomach achlohydria - give ascorbic acid

Other: incorrect dx, ongoing GI blood loss

Parenteral iron

Indications: Intolerance to oral, refractory to oral, GI disease (usually inflammatory)

New formulation is iron oxide coated in a carbohydrate

Iron deficit = % iron deficit x normal RBC volume

I.e. if the Hb is 75% of normal then it’s 0.25 x 27ml/kg (normal RBC volume) = 337.5 grams

The dose is then iron deficit + 1 gram = 1.4gm

How well did you know this?
1
Not at all
2
3
4
5
Perfectly
81
Q

A 65-year-old woman, established on haemodialysis for nine months, is maintained on erythropoietin 4,000 units twice a week intravenously. Previously her haemoglobin (Hb) had remained steady in the range of 105 – 115 gm/L. She now presents with a Hb of 89 gm/L on routine testing. There have been no changes to her erythropoietin dose or other medications.

Her mean corpuscular volume (MCV) is 84 fL [78-98],

her serum iron is 13 μmol/l [13-35], iron saturation is 18% [15-46],

serum ferritin is 630 μg/l [20-300] and her reticulocyte count is 30 x 109/L [12-100].

Which of the following is the most likely cause for her erythropoietin resistance?

A. Inflammation.

B. Pure red cell aplasia.

C. Vitamin B12 deficiency.

D. Iron deficiency.

E. Hyperparathyroidism.

A

A. Inflammation.

B. Pure red cell aplasia. – reticulocyte count normal

C. Vitamin B12 deficiency. – not macrocytic

D. Iron deficiency. – not microcytic, ferritin up so can’t say

E. Hyperparathyroidism. – not significant

How well did you know this?
1
Not at all
2
3
4
5
Perfectly
82
Q

What are the essential features of the anaemia of chronic disease?

A

Essentials

  • mild-mod normocytic or microcytic anaemia
  • normal-increased ferritin
  • normal-reduced transferrin
  • underlying chronic disease
How well did you know this?
1
Not at all
2
3
4
5
Perfectly
83
Q

What are the mechanisms of the anaemia of chronic disease?

A

Mechanism

  • EPO resistance
  • inflammatory cytokines diminishing RBC production
  • in the elderly: decreased EPO production due to reduced nephron mass and low level chronic inflammation. Normal serum iron
  • low serum iron in inflammatory states
How well did you know this?
1
Not at all
2
3
4
5
Perfectly
84
Q

What are the lab findings in the anaemia of chronic disease?

A

Lab findings

  • HCT rarely below 60% of baseline (except in renal failure)
  • normal or mildly reduced MCV
  • minimally changed reticulocyte count
  • low serum iron
  • low serum transferrin
  • normal to elevated serum ferreting (if <30, coexistent iron deficiency)
  • in organ failure and elderly: normal iron studies
  • if ferritin super elevated —> soluble transferrin receptor (high = iron deficiency
  • BMBx - absent iron staining, sequestration of iron in marrow macrophages but very rarely done.
  • ultimate test: Hb response to oral iron in setting of inflammation if iron deficiency is suspected.
How well did you know this?
1
Not at all
2
3
4
5
Perfectly
85
Q

What are the indications for EPO therapy?

A

EPO indicated in:

  • Hb <10 & anaemia due to RA, IBD, Hep-C, Zidovudine in HIV, myelosuppresive chemo w/ solid malignancy and palliative intent, CKD with GFR <60.
  • Aim for Hb 100-120, risk of VTE and ATE
How well did you know this?
1
Not at all
2
3
4
5
Perfectly
86
Q

What is the significance of marked erythroid hyperplasia in the bone marrow?

A

A sign of ineffective erythropoiesis (expansion of the erythroid compartment of the bone marrow that does not result in the adequate production of reticulocytes in the peripheral blood).

How well did you know this?
1
Not at all
2
3
4
5
Perfectly
87
Q

In a transfusion-dependent patient with idiopathic acquired sideroblastic anaemia, the most appropriate treatment for prevention of transfusional iron overload is:

A. vitamin C.

B. phlebotomy.

C. desferrioxamine.

D. ethylene-diamine tetra-acetic acid (EDTA).

E. pyridoxine.

A

C

How well did you know this?
1
Not at all
2
3
4
5
Perfectly
88
Q

Aside from being an anticoagulant for blood samples, what else is EDTA used for?

A

Lead poisoning chelation

How well did you know this?
1
Not at all
2
3
4
5
Perfectly
89
Q

What are the diagnostic essentials for sideroblastic anaemia?

A

Diagnostic essentials

  • ringed sideroblasts in the BM
  • elevated serum Fe+ and transferrin saturation
How well did you know this?
1
Not at all
2
3
4
5
Perfectly
90
Q

What is the mechanism and causes of sideroblastic anaemia?

A

Mechanism

Reduced ability to synthesise or incorporate heme into protophyrin IX —> accumulation of iron in RBC precursor mitochondria

Causes

  • subtype of MDS most commonly
  • chronic alcholism
  • lead poisoning
  • drugs: isoniazid, chloramphenicol
  • chronic infection/inflamattion
  • inherited: x-linked
How well did you know this?
1
Not at all
2
3
4
5
Perfectly
91
Q

What are the clinical and lab findings of sideroblastic anaemia?

A

Clinical findings

  • anaemia

Labs

  • sideroblastic MDS: Normal or slightly elevated MCV
  • all others: low
  • HCT of 20-30% (moderate anaemia)
  • elevated serum iron and transferrin saturation
  • smear: dimorphic RBCs (normal and hypo chromic), in lead poisoning: basophilic stippling, elevated serum lead
  • BMBx is diagnostic: marked erythroid hyperplasia with ringed sideroblasts (erythroid cells with iron deposits in mitochondria encircling nucleus) on Prussian blue iron stain.
How well did you know this?
1
Not at all
2
3
4
5
Perfectly
92
Q

What is the treatment of sideroblastic anaemia?

A

Treatment

  • occasionally transfusions
  • do NOT respond to EPO therapy
  • occasionally pyridoxine
  • remove offending toxin/drug in secondary forms
How well did you know this?
1
Not at all
2
3
4
5
Perfectly
93
Q

What is required for diagnosis of sideroblastic anaemia?

A

BMBx is diagnostic: marked erythroid hyperplasia with ringed sideroblasts (erythroid cells with iron deposits in mitochondria encircling nucleus) on Prussian blue iron stain.

How well did you know this?
1
Not at all
2
3
4
5
Perfectly
94
Q

What are the causes of B12 deficiency?

A

Dietary deficiency (rare)

Decreased production of intrinsic factor

  • Pernicious anemia
  • Gastrectomy

Helicobacter pylori infection Competition for vitamin B12 in gut

  • Blind loop syndrome
  • Fish tapeworm (rare)

Pancreatic insufficiency

Decreased ileal absorption of vitamin B12

  • Surgical resection
  • Crohn disease

Transcobalamin II deficiency (rare)

How well did you know this?
1
Not at all
2
3
4
5
Perfectly
95
Q

What are the essentials of diagnosis for B12 deficiency?

A
  • Macrocytic anemia.
  • Megaloblastic blood smear (macro-ovalocytes and hypersegmented neutrophils).
  • Low serum vitamin B12 level.
How well did you know this?
1
Not at all
2
3
4
5
Perfectly
96
Q

What is the physiology of B12 absorption and utilisation?

A

Physiology

B12 a cofactor in conversion of homocysteine to methionine, and for conversion of methylmalonylcoenzyme A (CoA) to succinyl CoA —> annulment of Okazaki fragments during DNA synthesis of erythroid progenitor cells

Dietary B12 is animal in origin —> —> absorbed by ileum —> liver stores 2-5mg while daily utilisation is 3-5mcg —> takes 3 years for iron deficiency to occur

How well did you know this?
1
Not at all
2
3
4
5
Perfectly
97
Q

What are the clinical findings of B12 deficiency?

A

Clinical findings

Slow onset mod-severe anaemia

May be asymptomatic

Later: leukopaenia and thrombocytoapaenia

Mucosal disturbance —> glossitis

Anorexia and diarrhoea

Neurology - peripheral nerves 1st —> parasthesia —> posterior column involvement —> loss of balance or proprioception or both —> later: dementia, neuropsychiatric abnormalities, decreased vibration and position sense

Non-hematologic complications can occur w/ NORMAL FBC

How well did you know this?
1
Not at all
2
3
4
5
Perfectly
98
Q

What are the lab findings in B12 deficiency?

A

Labs

Low serum B12 (cobalamin <170, symptomatic <100

Best confirmed with an elevated level of serum methylmalonic acid (can also be elevated i kidney disease

MCV >110 (but can have normal MCV, esp with Fe deficiency or thalassaemia)

Smear: Macro-ovalocytes (megaloblastic), hyper segmented neutrophils, reduced reticulocyte count, in severe cases: reduced WCC and PLTs (B12 deficiency affects all cell lines

Elevated LDH and indirect bilirubin due to intramedullary destruction of abnormal RBCs, similar to peripheral haemolytic anaemias

BMBx - erythroid hyperplasia, megaloblastic changes in erythroid precursors: abnormally large cell size, asynchronous maturation of the nucleus and cytoplasm (normal cytoplasm maturation, impaired DNA synthesis —> abnormal nucleus)

How well did you know this?
1
Not at all
2
3
4
5
Perfectly
99
Q

What is the ddx of B12 deficiency anaemia?

A

DDx

Other megaloblastic anaemias —> but B12 will be normal

MDS - different morphology, normal B12 and methylmalonic acid levels

How well did you know this?
1
Not at all
2
3
4
5
Perfectly
100
Q

What is the treatment of B12 anaemia?

A

Treatment

IM daily for first week, weekly for first month, monthly for life

Oral when corrected, even if pernicious anaemia as will be passively diffused if unable to be actively transported - continue indefinitely

Concurrent folic acid required due to intestinal mucosal disruption for several months

Hypokalemia may occur in first few days esp if anaemia severe

Neurology reversible if <6 months duration

How well did you know this?
1
Not at all
2
3
4
5
Perfectly
101
Q

A 46-year-old man with hypertension and previous end stage renal failure due to glomerulonephritis,

received a kidney transplant 15 years ago. His creatinine has been stable at 110-130 μmol/L

[60 - 120 μmol/L] for the last five years. During this time his immunosuppression has been unchanged

and consists of azathioprine 100 mg daily and prednisolone 5 mg daily.

His general practitioner is concerned about his reported mean corpuscular volume (MCV) of 106 fL

[80 – 96 fL]. Which of the following is the most likely cause?

A. Folate deficiency.

B. Azathioprine.

C. Myelodysplasia.

D. Haemolysis.

E. Hypothyroidism.

A

B

How well did you know this?
1
Not at all
2
3
4
5
Perfectly
102
Q

What is the differential diagnosis in B12 deficiency?

A

Other megaloblastic anaemias —> but B12 will be normal

MDS - different morphology, normal B12 and methylmalonic acid levels

How well did you know this?
1
Not at all
2
3
4
5
Perfectly
103
Q

What is the treatment of B12 deficiency anaemia?

A

Treatment

IM daily for first week, weekly for first month, monthly for life

Oral when corrected, even if pernicious anaemia as will be passively diffused if unable to be actively transported - continue indefinitely

Concurrent folic acid required due to intestinal mucosal disruption for several months

Hypokalemia may occur in first few days esp if anaemia severe

Neurology reversible if <6 months duration

How well did you know this?
1
Not at all
2
3
4
5
Perfectly
104
Q

A 42-year-old woman presents with several weeks of lethargy and is found to be clinically anaemic and mildly icteric. There is no history or clinical evidence of overt blood loss. She has a normal varied diet and is on no medication.

Her full blood count shows:

The blood film shows macrocytosis and a hypersegmented neutrophil

The most likely diagnosis is:

A. pernicious anaemia.

B. Evans’ syndrome (autoimmune haemolytic anaemia and thrombocytopenia).

C. aplastic anaemia.

D. hypothyroidism.

E. alcoholic liver disease.

A

A. Results in B12 deficiency –> megaloblastic anaemia

How well did you know this?
1
Not at all
2
3
4
5
Perfectly
105
Q

What are the causes of folate deficiency?

A

Dietary deficiency

Decreased absorption

  • Tropical sprue
  • Drugs: phenytoin, sulfasalazine, trimethoprim-sulfamethoxazole Concurrent vitamin B12 deficiency

Increased requirement

  • Chronic hemolytic anemia
  • Pregnancy
  • Exfoliative skin disease

Excess loss:

  • hemodialysis

Inhibition of reduction to active form

  • Methotrexate
How well did you know this?
1
Not at all
2
3
4
5
Perfectly
106
Q

What are the essentials for diagnosis of folate deficiency?

A
  • macrocytic anaemia
  • megaloblastic smear (macro-ovalocytes and hypersegmented neutrophils
  • normal serum B12 levels
  • reduced serum folic acid levels
How well did you know this?
1
Not at all
2
3
4
5
Perfectly
107
Q

What is the physiology of folate?

A

Physiology

  • real name: pteroylmonoglutamic acid
  • citrus and leafy greens
  • stored for 2-3 months
  • absorbed throughout entire GI tract
  • absorption impaired by drugs or full length mucosal disruption
  • increased requirements in pregnancy, haemolytic anaemia and exfoliative skin disease
How well did you know this?
1
Not at all
2
3
4
5
Perfectly
108
Q

What are the signs and symptoms of folate deficiency?

A

Signs and symptoms

  • anaemia
  • mucosal megaloblastic changes
  • NO neurological abnormalities
How well did you know this?
1
Not at all
2
3
4
5
Perfectly
109
Q

What are the lab findings in folate deficiency?

A

Macrocytic anemia.

Megaloblastic blood smear (macro-ovalocytes and hypersegmented neutrophils).

Normal serum vitamin B12

Low red blood cell folate (preferred as reflects stores over lifespan of RBC)

How well did you know this?
1
Not at all
2
3
4
5
Perfectly
110
Q

What is the ddx of folate deficiency?

A
  • B12 deficiency - check level
  • alcoholic nutritional deficiency —> no megaloblastic changes but TARGET cells
  • hypothyroidism assoc w/ pernicious anaemia
How well did you know this?
1
Not at all
2
3
4
5
Perfectly
111
Q

What is the treatment of folic acid deficiency?

A

Tx

Daily PO folic acid, similar response as in B12 replacement

  • MUST check B12, large doses of folic acid will correct bloods but will allow ongoing neurologic damage.
How well did you know this?
1
Not at all
2
3
4
5
Perfectly
112
Q

Graft-versus-host disease is a major complication of allogeneic stem cell transplantation for haematological malignancy in adults.

Which one of the following has the least influence on the risk of developing graft-versus-host disease?

A. Age of recipient.

B. Donor-recipient ABO blood group incompatibility.

C. In vitro T cell depletion of the graft.

D. Type of HLA-matched donor (sibling versus unrelated).

E. Donor-recipient HLA mismatch.

A

B. Donor-recipient ABO blood group incompatibility.

How well did you know this?
1
Not at all
2
3
4
5
Perfectly
113
Q

The major cause of death in patients more than six months following cardiac transplantation is:

A. graft-versus-host reaction.

B. opportunistic infection.

C. rejection.

D. malignancy.

E. coronary artery disease.

A

E

How well did you know this?
1
Not at all
2
3
4
5
Perfectly
114
Q

A 42-year-old man with acute myeloid leukaemia undergoes allogeneic stem cell transplantation from a human leucocyte antigen (HLA)-matched sibling. He is progressing well for the first 10 days, but then complains of abdominal pain and leg swelling.

He is noted to have marked ascites, moderate peripheral oedema and jaundice.

Liver function tests are as follows:

An abdominal ultrasound confirms large volume ascites with an enlarged liver (span 18 cm). The spleen is normal in size.

The most likely explanation for his condition is:

A. graft-versus-host disease.

B. cytomegalovirus hepatitis.

C. hepatosplenic candidiasis.

D. portal vein thrombosis.

E. veno-occlusive disease.

A

E. veno-occlusive disease.

How well did you know this?
1
Not at all
2
3
4
5
Perfectly
115
Q

Which of the following is the greatest cause of mortality occurring more than 100 days following allogeneic peripheral blood cell stem transplantation for acute leukaemia?

A. Hepatic veno-occlusive disease.

B. Graft-versus-host disease.

C. Transplantation-related lung injury.

D. Transplantation-related infection.

E. Recurrent leukaemia.

A

E

How well did you know this?
1
Not at all
2
3
4
5
Perfectly
116
Q

What are the early post hematopoetic stem cell transplant complications?

A

Mucositis (most common)

Haemorrhagic cystitis

Prolonged and severe pancytopaenia

Infections

CMV

EBV

Viral hepatitis

Acute GVHD

Graft failure

Transplant related lung injury

Hepatic veno-occlusive disease

Transplant related MAHA

How well did you know this?
1
Not at all
2
3
4
5
Perfectly
117
Q

What is the most common early HSCT transplant complication?

What causes it?

How do you treat it?

A

Mucositis

  • most common short-term complication
  • etoposide and MTx
  • can be oropharyngeal and intestinal
  • intubation if supraglottic
  • may need TPN
  • topical and systemic analgesia and opioids
How well did you know this?
1
Not at all
2
3
4
5
Perfectly
118
Q

When does severe pancytopaenia occur in HSCT? How is it managed? When do infections occur?

A

Prolonged & severe pancytopaenia

  • common, lasts up to 4 weeks
  • empiric anti fungal tx if persistent unexplained fever in setting of use of broad-spectrum abc
  • antiviral prophylaxis usually given
  • serious infections (pneumonia, bacteraemia, fungaemia, viraemia) in up to 50% - more common in matched unrelated donors (MUDS) than auto or sibling allografts.
  • recombinant growth factors (filgrastim) started 24-72 hours post stem cell infusion reduce neutrophil recovery time
  • severe thrombocytopaenia —> transfuse <10, or 50 if surgery required.
  • severe anaemia —> transfuse, sometimes EPO
How well did you know this?
1
Not at all
2
3
4
5
Perfectly
119
Q

What is the timeline of infection in HSCT?

A

Infections

Early (0-30 days) - mucosal and skin injury —> aerobic bacteria (esp coag -ve staph, viridans step, gram -ve’s candida and HSV).

Mid 1-3 months: T-cell dysfunction, hypogammaglobulinaemia —> encapsulated bacteria (pneumococcus, H.influenzae), viruses (CMV, PHP), molds and candida

Later 3-12 months - t-cell reconstitution, chronic GVHD —> encapsulated bacteria, CMV, PJP, HZV, EBV and hepatitis

How well did you know this?
1
Not at all
2
3
4
5
Perfectly
120
Q

What causes CMV and EBV infection in HSCT?

A

CMV

  • due to impaired viral immunity in first year, acute or chronic GVHD
  • CMV positivity = higher peritransplanatation mortality rate. - treated prophylactically and preemptively to prevent pneumonitis and viraemia but can still occur

EBV

  • most common in patients who are EBV naive, in patients receiving T-cell depleted grafts, or receiving ATG for in-vivo t-cell depletion.
  • causes posttrantplant lymphoproliferative disorder (PTLD) in HSCT, t-cell depleted grafts, and those undergoing intensive tx for GVHD
How well did you know this?
1
Not at all
2
3
4
5
Perfectly
121
Q

What role does viral hepatitis play in HSCT?

A

Viral hepatitis

  • third most common cause of liver disease in transplant patients
  • can reactivate HBV at 3-6 months due to impaired cellular immunity
  • HBsAg +ve = prophylactic antiviral tx pre-chemotherapy and for 3/12 after.
  • if -ve, need vaccination before transplant
  • HCV has little impact but is a risk factor for hepatic veno-occlusive disease and GVHD
  • faster progress of HCV to fibrosis and cirrhosis, decompensation and malignancy in HSCT. Third leading cause of late deaths after transplant - need definitive management after 6 months and no evidence of GVHD.
How well did you know this?
1
Not at all
2
3
4
5
Perfectly
122
Q

What causes GVHD in HSCT? Who gets it? How do you reduce it?

A

Donor immunocompetent T-cells and NK cells recognise host antigen as foreign —> immune reaction

  • mainly in allograft setting (not auto or syngeneic (twin)
  • severity of GVHD inversely related to risk of relapse due to relationship with graft-versus-leukaemia effect.
  • reduction achieved by optimising donor and graft type and post-transplant immunosuppresion.
  • least with peripheral blood HSCT, worst with umbilical cord HSCT
How well did you know this?
1
Not at all
2
3
4
5
Perfectly
123
Q

When does acute GVHD occur, what does it affect, what are the risk factors, pathogenesis and treatment?

A
  • common, within first 100 days
  • skin, mucosa, gut, and liver
  • erythematous macular skin rash —> blistering (like burns), severe abdominal pain, profound diarrhoea, hyperbilirubinaemia.
  • Stage 1 skin up to stage IV (systemic). III-IV have a bad prognosis.
  • RFs: HLA mismatch, MUD grafts, grafts from a parous female donor, older age
  • pathogenesis: acute cytokine storm (TNF and IL-1) released due to damaged host tissue from induction —> increased MHC expression —> recognition of minor HLA differences by T-cells —> proliferation and cytokine release —> further recruitment of t-cells and macrophages —> release of TNF and IL-1 —> vicious cycle of inflammation and tissue damage
  • prophylaxis better than treatment: graft t-cell depletion (but increased risk of graft failure and rate of relapse due to graft vs tumour effect
  • prevention: CNi (renal toxicity) + MTx (but mucositis), sirolimus and MMF have less toxicity. Can also use gut decontamination with metro, IVIG or less intensive regiment
  • high dose steroids and ATG does not work.
How well did you know this?
1
Not at all
2
3
4
5
Perfectly
124
Q

Who gets chronic GVHD?

What are the risk factors?

What are the clinical features

How do you manage it?

A
  • 40-80% of long term survivors, incidence rising as older patients get transplants
  • RFs: peripheral blood transplants, mismatch or unrelated donors, second transplant, donor leukocyte infusions
  • greatest risk for chronic GVHD is acute GVHD
  • 2-12 months post transplant - skin, eyes, mouth, liver, fascia, any organ
  • chronic lichenoid skin changes, dry eyes and mouth, impaired ROM from fibrosis of dermis and fascia. Resembles scleroderma or other autoimmune diseases
  • immunosuppression with steroids, tacrolimus and MMF are mainstays, hydroxychloroquine works well too.
  • major cause of death: profound immunodeficiency, need prophylaxis against encapsulated organisms
  • patients with frequent infections and low Ig levels should get IVIG
How well did you know this?
1
Not at all
2
3
4
5
Perfectly
125
Q

What are the risk factors for, and treatment of HSCT graft failure?

A

Graft failure

  • associated with HLA mismatch, more frequent in MUD grads
  • other RFs: aplastic anaemia, t-cell depletion, infusion of lower number of stem cells (cord blood), nonmyeloablative transplants, GVHD, splenomegaly
  • poor graft function can be treated with growth factors (G-CSF or GM-CSF), and EPO.
  • Failure: second stem cell infusion
How well did you know this?
1
Not at all
2
3
4
5
Perfectly
126
Q

What are the early pulmonary complications of HSCT transplant? Treatment?

A

Transplantation-relation lung injury (TRLI)

  • acute inflame response —> severe lung injury
  • allogenic transplant
  • early tx w/ corticosteroids and etanercept (anti-TNF) reduces extent

Interstitial pneumonitis (usually CMV) is frequently fatal - reduced w/ anti-infective prophylaxis and CMV-ve blood products (leukodepletion, CMV -ve donors), tx w/ ganciclovir or foscarnet + IvIg

Autograft - diffuse alveolar haemorrhage

RTx or pulm toxins, MTx or carmustine

How well did you know this?
1
Not at all
2
3
4
5
Perfectly
127
Q

What is Sinusoidal Obstruction Syndrome? What are the risk factors? Pathogenesis? Treatment? Outcomes?

A

Hepatic veno-occlusive disease (Sinusoidal Obstruction Syndrome)

  • very common, potentially lethal
  • 10-60% of patients, 50% of post-transplant deaths
  • weight gain, tender hepatomegaly, jaundice and ascites —> fulminant multiorgani failure
  • 8-10 days post inductions
  • RFs: prior liver damage, high levels of busulphan, >10-12 Gy total body irradiation, heavy induction, C282Y positivity of haemochromatosis gene
  • pathology: elevated TNF —> sinusoidal endothelial damage —> sloughs —> obstructs hepatic circulation —> centrilobular hepatic injury and portal hypertension.
  • Prevention is best treatment: avoid fludarabine or cyclophosphamide, use nonmyeloablative regiment
  • Ursofalk reduces risk of this and grades III-IV GVHD
  • Treatment is supportive, with defibrotide which is antithrombotic and fibrinolytic (avoid heparin and TpA)
How well did you know this?
1
Not at all
2
3
4
5
Perfectly
128
Q

What are the late complications of HSCT transplant?

A
  • organ toxicity from chemo
  • posttransplanation immunosuppression
  • Chronic GVHD
  • endocrine disease and heart failure
  • increased risk of malignancy years later: acute leukaemia’s, solid tumours, MDS, disease and regimen dependent, increased prevalence after total-body irradiation
  • late infections (months), usually in assoc w/ GVHD or GVHD therapy
  • Vaccines: pneumococcus, H.influenzae, Hep-B, polio, ADT, influenza - recommence at 18 months

Ocular

  • cataract formation
  • dry eyes due to chronic GVHD

Bronchiolitis obliterans

  • HSCT recipients, fatality rate 50%
  • no response to steroids

MSK

  • OP, and avascular necrosis
  • bisphosphonates

Neuropsych disorders

  • usually due to cranial irradiation
How well did you know this?
1
Not at all
2
3
4
5
Perfectly
129
Q

What are the long-term immune effects of HSCT transplant?

A
  • host immunity suppressed months-years
  • worse after allograft
  • related to severity of induction, acute GVHD, ongoing immunosuppression for GVHD
  • complete reconstitution may take years due to less active (or even absent) thymic function the older you get.
How well did you know this?
1
Not at all
2
3
4
5
Perfectly
130
Q

Which HLA loci must be matched in allo-HSCT?

A

Donor must be matched with the patient (recipient) at the HLA loci (HLA A, B, C, DR) that specify major histocompatibility antigens.

How well did you know this?
1
Not at all
2
3
4
5
Perfectly
131
Q

Where are donor stem cells collected from in allo-HSCT?

A
  • umbilical cord blood units may also be used.
  • bone marrow
  • more commonly through leukopheresis of the blood after mobilizing hematopoietic stem cells from the bone marrow with filgrastim (G-CSF).
How well did you know this?
1
Not at all
2
3
4
5
Perfectly
132
Q

What is the initial period of pancytopaenia following induction and HSCT-transplant?

A

There is a period of pancytopenia in the gap between the effect of the chemotherapy given to the patient and the time it takes the infused hematopoietic stem cells to grow into bone marrow, usually 10–14 days.

How well did you know this?
1
Not at all
2
3
4
5
Perfectly
133
Q

What is the major cause of morbidity and mortality in allo-HSCT?

How is it managed?

A

GVH is the major cause of morbidity and mortality during an allogeneic SCT. Immunosuppression must be given during allogeneic stem cell transplantation to reduce the incidence and severity of GVH reaction.

CnI plus MTx

How well did you know this?
1
Not at all
2
3
4
5
Perfectly
134
Q

Assuming no complications, when does immunosuppression post HSCT transplant cease?

A

In most cases of allogeneic stem cell transplantation, the immunosuppression can be tapered and discontinued 6 or more months after transplantation.

How well did you know this?
1
Not at all
2
3
4
5
Perfectly
135
Q

What is the graft-versus malignancy (GVM) effect?

How does this relate to graft-versus-host disease?

What is the treatment implication?

A

Residual cancer cells can be recognized as foreign by the donor immune system and killed.

Intensity of GVHD correlates with GVM (rate of graft failure is inversely proportional to severity of GVHD)

Reducing the intensity of the induction regimen, relying for cure more on the GVM effect than the myeloablation. Allo-SCT is now being offered up to age 60 in some cases.

How well did you know this?
1
Not at all
2
3
4
5
Perfectly
136
Q

When is allo-HSCT curative?

A

Acute leukaemias (AML, ALL)

MDS

CML (refractory to TKIs)

Severe aplastic anaemia

How well did you know this?
1
Not at all
2
3
4
5
Perfectly
137
Q

What is the main dose-limiting toxicity of chemotherapy?

How can you improve it?

A

Bone marrow failure

Autograft - G-CSF is given –> stem cells are collected from peripheral blood pre treatment –> induction Ctx (to kill cancer) –> reinfused post

How well did you know this?
1
Not at all
2
3
4
5
Perfectly
138
Q

How long does the severe pancytopaenia post induction and HSCT transplant last? How is it managed?

A

7-10 days

Transfusion and antibiotics

How well did you know this?
1
Not at all
2
3
4
5
Perfectly
139
Q

When is autografted stem cell transplant used?

A

DLBCL recurrence after chemo but still responsive to chemo

Responsive relapsed Hodgkin’s lymphoma

Recurred testicular germ cell tumours

How well did you know this?
1
Not at all
2
3
4
5
Perfectly
140
Q

What is the role of autologous stem cell transplant in mantle cell lymphoma and multiple myeloma?

A

Autologous stem cell transplantation is currently part of the standard of care for the treatment of mantle cell lymphoma and multiple myeloma, based not on curative potential, but the prolongation of remission and overall survival.

How well did you know this?
1
Not at all
2
3
4
5
Perfectly
141
Q

What are acanthocytes?

What causes them?

A

Acanthocytes arise from either of two mechanisms. Alterations in membrane lipids are seen in abetalipoproteinemia and liver dysfunction. Alteration in membrane structural proteins are seen in neuroacanthocytosis and McLeod syndrome.

In liver dysfunction, apolipoprotein A-II deficient lipoprotein accumulates in plasma causing increased cholesterol in RBCs. This causes abnormalities of membrane of RBC causing remodeling in spleen and formation of acanthocytes.

How well did you know this?
1
Not at all
2
3
4
5
Perfectly
142
Q

What is leucodepletion?

On what is it performed?

What are the benefits?

A


Leucodepletion is the removal of white blood cells from a blood component.

Applies to whole blood, red cells and platelets.

Benefits:

  • Reduction in platelet refractoriness


- Reduction in febrile non-haemolytic transfusion reactions. (FNHTR)


- Reduction in CMV transmission risk


- Improved chance of finding an organ transplant match if required


- Reduction in storage lesion effect


- Possible reduction in transfusion associated graft vs host disease (TA-GVHD) risk

How well did you know this?
1
Not at all
2
3
4
5
Perfectly
143
Q

Leucodepletion by filtration of blood for transfusion is most likely to reduce the risk of which one of the following transfusion-associated events in the transfused recipient?

A. Hepatitis C.

B. Citrate toxicity.

C. Cytomegalovirus infection.

D. Malaria.

E. Anaphylactic reaction.

A

C

How well did you know this?
1
Not at all
2
3
4
5
Perfectly
144
Q

A 48-year-old previously well man is receiving a transfusion of four units of packed red cell concentrate via a peripheral intravenous line after presenting with melaena secondary to a bleeding duodenal ulcer. He has not been previously transfused. Admission biochemistry (including urea, electrolytes and creatinine) is normal and pre-transfusion blood counts are normal apart from anaemia (haemoglobin 64 g/L [135-170]). Less than five minutes after transfusion of the third unit of blood is commenced, he complains of feeling very unwell and abruptly develops fever, chills, rigors and profound hypotension.

In the absence of ABO, Rhesus or minor blood group donor-recipient incompatibility, the most likely diagnosis is:

A. bacterial contamination of the transfused blood.

B. transfusion-associated graft-versus-host disease.

C. electrolyte disturbance.

D. citrate toxicity.

E. reaction to plasticiser in infusion bag.

A

B

How well did you know this?
1
Not at all
2
3
4
5
Perfectly
145
Q

What blood can an AB- person receive?

A

O, A, and B negative blood only

(Negative can only receive negative, never positive)

How well did you know this?
1
Not at all
2
3
4
5
Perfectly
146
Q

What blood can an AB+ person receive?

A

A, B and O all positive or negative

Positive blood groups can receive negative matches.

How well did you know this?
1
Not at all
2
3
4
5
Perfectly
147
Q

What blood can an A+ person receive?

A

A-, A+ or O +/-

(Positive can receive negative or positive)

(O bloods do not have A or B antigens)

How well did you know this?
1
Not at all
2
3
4
5
Perfectly
148
Q

What blood can an A- person receive?

A

A- or O- only

Negative can only receive negative

O does not have A or B antigens but still carries rhesus status.

How well did you know this?
1
Not at all
2
3
4
5
Perfectly
149
Q

What blood can an B+ person receive?

A

B +/-, O+/-

Positive can receive positive or negative

O does not carry A or B antigens, only rhesus status

How well did you know this?
1
Not at all
2
3
4
5
Perfectly
150
Q

What blood can an B- person receive?

A

B- or O- blood.

Negative can only receive negative

O has no A or B antigens, only rhesus.

How well did you know this?
1
Not at all
2
3
4
5
Perfectly
151
Q

What blood can an O- and O+ person receive?

A

O- can only receive O-

O+ can receive both O+ and O-

No A or B can be given because the recipient will have antibodies

Negative can only receive negative

O does not have A or B antigens

How well did you know this?
1
Not at all
2
3
4
5
Perfectly
152
Q

How much will 1 unit of PRBCs raise the HCT by?

A

Approx 4%

How well did you know this?
1
Not at all
2
3
4
5
Perfectly
153
Q

When are frozen red blood cells used?

A

Rare blood types

Occasionally - severe leukoagglutinin reactions or anaphylactic reactions

How well did you know this?
1
Not at all
2
3
4
5
Perfectly
154
Q

Why are the A and B antigen important in blood types?

A

If you lack one, you have an IgM isoantibody to it (isoagglutinin) –> complement gets activated –> rapid intravascular lysis.

How well did you know this?
1
Not at all
2
3
4
5
Perfectly
155
Q

Why is type O negative blood used in emergencies?

What needs to be done to it?

A

It does not have A or B antigens, and thus can be given to anyone.

PRBCs can only be given (the plasma has to be separated out) because the donor plasma contains Anti-A or Anti-B antibodies.

How well did you know this?
1
Not at all
2
3
4
5
Perfectly
156
Q

Why is the Rhesus antigen important in blood typing?

A

Rh D antigen is highly immunogenic

Anyone who receives D+ blood when they are D- will subseuqnetly make highly active anti-D antibodies that cause severe lysis of subsequent transfusions of D+ red cells or to a future rhesus positive baby

How well did you know this?
1
Not at all
2
3
4
5
Perfectly
157
Q

Why is anti-D given to rhesus-negative mothers?

A

Stops allo-immunisation by destroying any Rh-D positive fetal erythrocytes that get through to the maternal system before the mother can make her own antibodies.

Prevents haemolytic disease (rhesus disease) of the newborn.

Okay if you have an RH+ mother and an Rh- fetus

How well did you know this?
1
Not at all
2
3
4
5
Perfectly
158
Q

What causes haemolytic transfusion reactions?

A

Acute
ABO incompatibility - isoagglutinin mediated
Mostly due to clerical errors and mislabelled specimens

Rapid intravascular haemolysis - most severe usually when the patient is under anaesthesia

Delayed

Minor RBC antigen discrepancies, less severe, IgG alloantibody mediates

5-10 days post transfusion

Most common antigens are duffy, Kidd, Kell and Rh C and E

SSx

Fever, chills, backache and headache

Apprehension, dyspnoea and hypotension

Generalised bleeding and oliguria

Severe: DIC, ARF due to ATN

Delayed: usually w/o SSx

Tx - hydration, maybe forced diuresis with mannitol

How well did you know this?
1
Not at all
2
3
4
5
Perfectly
159
Q

What is a leukoaggluttinin transfusion reaction?

A

Leucocyte rich blood products esp platelets cause - happens in 1-2% of transfusion

Fevers and chills within 12 hours post

No haemolysis

Responds to panadol and dipenhydramine, steroids

Less of an issue now with leuco depletion

Different to a hypersensitivity reaction

How well did you know this?
1
Not at all
2
3
4
5
Perfectly
160
Q

What is the cause of a blood product hypersensitivity reaction?

A

Exposure to allogeneic plasma proteins (not leukocytes)

Anaphylactic shock may develop in IgA deficient patients because of IgA antibodies in the plasma component of the donor product.

May require washed or frozen PRBCs next time.

How well did you know this?
1
Not at all
2
3
4
5
Perfectly
161
Q

Under what circumstances does contamination of blood products occur?

A

Platelets especially because they cannot be refrigerated

Gram positive contaimination will cause fever and bacteraemia but rarely sepsis.

Gram negative will cause septic shock, acute DIC and ARF due to transfused endotoxin - usually fatal.

Reduced by

Aseptic technique

Diverting first few ml of donated blood

Single donor products (instead of pooled)

POC bacterial screening to discard dodgy units

How well did you know this?
1
Not at all
2
3
4
5
Perfectly
162
Q

Which blood products can transmit viral diseases?

A

All blood products can.

Risk of post-transfusion viral hepatitis is very low.

Leukodepletion has greatly reduced CMB transmission, possibly equivalent to CMV-screened negative blood products.

How well did you know this?
1
Not at all
2
3
4
5
Perfectly
163
Q

What is tranfusion graft-versus-host disease?

A

Usually fatal, fever, rash, diarrhea, hepatitis, lymphadenopathy and severe pancytopaenia

Usually in patients with immune defects, malignant lymphoproliferative disorders, chemo or immunotherapy, immunosuppression, old patients getting cardiac surgery

HIV alone does not increase risk

Leukodepletion doesn’t avoid, need to irradiate the blood products if high risk.

How well did you know this?
1
Not at all
2
3
4
5
Perfectly
164
Q

What is TRALI? What causes it and how does it present?

A

Transfusion-related Acute Lung Injury

Allogeneic antibodies in donor plasma bind to recipient leukococyte antigens including HLA, granulocyte and monocyte specific.

Definition: Non-cardiogenic pulmonary oedema after a blood product transfusion without other explanation. Surgical and critically ill patients most susceptible.

Reduced with male only plasma, treated with supportive care.

How well did you know this?
1
Not at all
2
3
4
5
Perfectly
165
Q

At what platelet level is there a higher risk of spontaneous bleeds?

Life threatening bleeding?

When do you transfuse?

What do you aim for?

A

<80

<5

Given when less then 10

In thrombocytopaenia pre-op patients, goal is to get the platelet count >50

How well did you know this?
1
Not at all
2
3
4
5
Perfectly
166
Q

What are the causes of poor platelet response post transfusion?

A

Fever, sepsis

Splenomegaly

Lagre habitus

Low platelet dose in transfusion

Platelet alloimmunisation from prior transfusion
Prior pregnancy

Prior organ transplantation

May get alloantibodies directed at HLA antigens - can get HLA matched antigens if this is the case.

How well did you know this?
1
Not at all
2
3
4
5
Perfectly
167
Q

In the rare circumstance that you would have to get a granulocyte transfusion (ie failed G-CSF), what needs to happen?

A

Must be ABO matched

Must be irradiated to prevent lymphocyte proliferation and prevent GVHD without funcitonal harm to the granulocytes or containing platelets.

BUT not commonly used anyway

168
Q

What are febrile non-haemolytic transfusion reactions mostly due to?

A

Recipient has antibodies to donor antigens on leukocytes (granulocytes) due to previous transfusion or pregnancy.

Leucodepletion reduces febrile non-haemolytic transfusion reactions as a result.

169
Q

A 19-year-old woman presents with recent-onset of right leg swelling and pleuritic chest pain and is found to have iliofemoral thrombosis on Doppler ultrasound examination. Results of a ventilation-perfusion lung scan indicate a high probability for pulmonary emboli. She had been taking the oral contraceptive pill for the last three years but has now ceased. She is a non-smoker. There is no significant medical history and no known family history of venous thromboembolism.

Which one of the following investigations for an underlying hypercoagulable state is most likely to be affected by the presence of the extensive thrombosis?

A. Antiphospholipid antibody screen.

B. Antithrombin level.

C. Factor V and prothrombin genotype assessment.

D. Activated partial thromboplastin time (APTT).

E. Full blood count including blood film.

A

B

170
Q

What is the mechanism of ITP?

A

Antiplatelet antibody to glycoprotein IIb/IIa or Ib/IX on the platelet membrane –> platelet destruction –> mucocutaneous bleeding

171
Q

What is the aetiology of ITP?

A

Primary / idiopathic

Medication related (Thiazides, penicillin, GPIIB/IIIa inhibitors (ie tirofiban, Heparin)

Hep-C

HIV

172
Q

What is the mechanism of ITP in HIV? Hep-C?

A

HIV: Direct suppression of platelet production

Hep-C: Cirrhosis related splenomegaly

173
Q

What are the clinical findings in ITP?

A

Mucocutaneous bleeding: nosebleeds, gingiva

Spontaneous bruising

Usually at PLT counts <20-30

Disease specific findings if HIV or Hep-C

174
Q

What are the lab findings in ITP?

A

Isolated thrombocytopaenia

Anaemia if they’ve bled

Exclude HIV and Hep-C

BMBx for cytopaenias if over 60 or failed to respond to ITP therapy

Megakaryocyte abnormalities are not ITP

175
Q

What is the treatment of ITP?

A

Treat when PLT <20-30 or significant bleeding

Mainstay: short course of corticosteroids however most relapse. Plus or minus IVIG or anti-D (WinRho)

Persistent plt count <30 after roids: second line treatment (anti D, IviG are both temporary), rituxmiab works in up to 50%. Splenectomy an option

176
Q

Name the thrombopoietin receptor agonists

A

Eltrombopag

Romiplostin

Used in chronic ITP for people who have not responded to corticosteroids, IvIG or splenectomy or in cirrhosis due to hep-C. Must be taken indefintely

177
Q

What vaccinations should be administered pre-splenectomy?

A

Patients should receive pneumococcal, Haemophilus influenzae type b, and meningococcal vaccination at least 2 weeks before the procedure

178
Q

What are the goals and management of pregnancy-associated ITP?

A

The goal of management of pregnancy-associated ITP is a platelet count of 10,000–30,000/mcL in the first trimester, >30,000/mcL during the second or third trimester, and >50,000/mcL prior to cesarean section or vaginal delivery. Moderate-dose oral prednisone or intermittent infusions of IVIG are standard.

179
Q

How is the thrombocytopaenia of HIV and Hep-C managed?

A

Treatment of either infection improves platelet count in most cases

Refractory thrombocytopenia may be treated with infusion of IVIG or anti-D (HIV and hepatitis C virus), splenectomy (HIV), or interferon-alpha or eltrombopag (hepatitis C virus, including eradication).

Treatment with corticosteroids is not recommended in hepatitis C virus infection.

180
Q

A 30-year-old man has previously undergone splenectomy (six months prior) for chronic idiopathic thrombocytopenic purpura (ITP). Apart from occasional bruising, there are no haemorrhagic symptoms.

His full blood examination shows:

_

Repeated autoimmune and viral serology remain negative.

The most appropriate management of his thrombocytopenia is:

A. danazol.

B. immunoglobulin.

C. prednis(ol)one.

D. azathioprine.

E. observation.

A

E. Platelet count gets treated when <30 in refractory cases or if bleeding.

181
Q

A 26-year-old previously well woman on no medication presents with the recent-onset of easy bruising. She has two small oral mucosal haemorrhages, scattered bruises and petechiae around the ankles but no other abnormalities on examination.

Human immunodeficiency virus (HIV) antibody test is negative.

The full blood count shows:

Prothrombin time-international normalised ratio (PT-INR) and activated partial thromboplastin time (APTT) are normal and bone marrow examination is also normal except for an increase in megakaryocytes.

The most appropriate initial therapy is:

A. intravenous gamma globulin.

B. oral tranexamic acid.

C. platelet transfusion.

D. oral prednis(ol)one.

E. oral danazol.

A

D

182
Q

A 23-year-old woman presents with several days history of petechiae of the skin and oral mucosa. There is no history of infection or medication apart from the oral contraceptive pill. The patient is well and examination is unremarkable except for the petechiae.

Investigations reveal:

Which one of the following is the most appropriate treatment?

A. Oral prednis(ol)one.

B. Intravenous immunoglobulin.

C. Platelet transfusion.

D. Danazol.

E. Splenectomy.

A

A

183
Q

What is the cause of red cell aplasia?

What are the characteristics of the anaemia?

What is the treatment?

A

Idiopathic and autoimmune (mediated by T-lymphocytes or IgG antibody to erythroid precursors.

Viral: (parvo, herpes, HIV and hepatitis)

Thymoma

Other malignancy

Drugs: EPO (alpha aka Eprex), MMF

Severe and normochromic with absent retics

BMBx: All non- erythroid elements are present and normal, but erythroid precursors are markedly reduced or absent. In some cases, chest imaging studies will reveal a thymoma.

Combination of antithymocyte globulin and a calcineurin inhibitor (cyclosporine or tacrolimus)—similar to therapy of aplastic anemia.

184
Q

What is the mechanism of EPO related red-cell aplasia?

A

Neutralising antibodies, and will cross-react with all currently available erythropoietic agents (epoetin alfa and beta, and darbepoetin alfa), in addition to endogenous EPO.

185
Q

In which conditions are spherocytes found?

A

Spherocytes are found in hereditary spherocytosis and autoimmune hemolytic anemia.

186
Q

What is a ringed sideroblast?

A

Ring sideroblasts are named so because of the arrangement of the iron granules in a ring form in mitochondria around the nucleus of an erythroid precursor

The body has iron available but cannot incorporate it into hemoglobin, which red blood cells need to transport oxygen efficiently.

187
Q

What are the diagnostic essentials of myelodysplastic syndrome?

A
  • cytopaenia with hyper cellular BM (but not more than 20% blasts otherwise it’s AML)
  • morphologic abnormalities in two or more cell lines
188
Q

What are the causes of myelodysplastic syndrome?

A

Idiopathic / cytotoxic CTx, RTx or both

Benzene

Red cell aplasia and Fanconi’s anaemia can transform to MDS

189
Q

What is the classification of stages of MDS?

A

Refractory anaemia - no excess bone marrow blasts, may or may not have ringed sideroblasts

Refractory anaemia w/ excess blasts

Chronic myelomonocytic leukaemia - MDS + peripheral blood monocytosis

Prognosis worsens with each.

190
Q

What is 5q- syndrome?

A

Myelodysplasia with:

5q- syndrome (indolent disease)

Refractory anaemia with loss of long arm of chromosome 5

Good prognosis

191
Q

How is the severity and survival of myelodysplasia ranked?

A

IPSS - survival and rate of progression AML

Survival related to:

Percentage of bone marrow blasts 0-10% mild, 21-30% is poor

Karyotype: Normal, Y-, 5q-, 20q- = good

Bad: Cr 7 (monosomy 7), or if there are 3 or more abnormalities

Cytopaenias - None or 1 cell line is good, 2 or 3 is bad.

192
Q

What are the clinical findings in MDS?

A

Clinical findings

Age >60

Incidental finding or fatigue/infection/bleeding due to bone marrow failure

Fever, weight loss, general debility

Splenomegaly, pallor and bleeding

193
Q

What are the lab findings in myelodysplasia?

A

Marked anaemia w/ normal or elevated MCV

Low WCC and neutropenia

Smear: Macro-ovalocytes, bilobed neutrophils (Pelger Huet abnormality), myeloid left shift with small numbers promyelocaytes or blasts, normal or reduced hypo granular platelets

BMBx: Erythroid hyperplasia, megaloblastosis: nuclear budding, multinucleate erythroid precursors

Left shifted myeloid series, characteristic: dwarf megakaryocytic with a unloved nucleus

Prussian blue stain: ringed sideroblasts

194
Q

How do you differentiate myelodysplasia from MDS?

A

AML: MDS has <20% blasts, AML has >20%

195
Q

What is the treatment of myelodysplasia?

A

EPO if anaemic if EPO levels are low

G-CSF may help response to EPO.

Highest transfusion requirements = least likely to respond

Lenalidomide for transfusion dependent anaemia due to MDS esp for 5q- syndrome (less success w/o it). SEs: neutropenia, thrombocytopaenia, VTE - must use aspirin prophylaxis. Not useful for other cell lines.

Iron chelation if transfusion dependent.

High risk - azacitidine

196
Q

What is the prognosis of myelodysplasia

A

Ultimately fatal, cure rates of 30-60%

5q- 5 year survival over 90% or low risk disease

Excess blasts or CMML = higher chance of AML and short survival w/o Allo SCT.

197
Q

What are the diagnostic essentials of hairy cell leukaemia?

A

Pancytopenia.

Splenomegaly, often massive.

Hairy cells present on blood smear and especially in bone marrow biopsy

198
Q

What is hairy cell leukaemia?

A

Rare mature b-lymphocytosis with hairy cytoplasmic projections.

199
Q

What are the signs and symptoms of hairy cell leukaemia?

A

Middle aged men, 55 years 5:1 M:F

Fatigue

Sx related to markedly enlarged spleen

Recurrent infection inc mycobacterial infections

Hepatomegaly in 50%

No lymphadenopathy

200
Q

What are the lab findings in hairy cell leukaemia?

A

Pancytopaenia (hallmark)

Anaemia (universal), thombocytopaenia and neutropenia

Smear: ‘hairy cells’ B-lymphocytes with cytoplasmic projections

BMBx: dry tap

Splenic infiltrate of red pulp with hair cells (lymphoma it’s the white pulp)

Immunophenotyping: CD11c, CD20, Cd22, CD25, CD103, and CD123

201
Q

What is the treatment of hairy cell leukaemia?

A

IFN-a

Cladribine (purine analogue)

Pentostatin

202
Q

What is the prognosis of hairy cell leukaemia?

A

Good - >95% of patients are alive longer than 10 years

If secondary to CLL - worse prog

203
Q

What are the diagnostic essentials of Waldenstrom’s macroglobulinaemia?

A

Monoclonal IgM paraprotein

BM infiltration by plasmacytic lymphocytes

Absence of lytic bone disease

204
Q

What is Waldenstrom’s macroglobulinaemia?

A

IgM hypergammaglobulinaemia in setting of NHL.

B-lymphocytes that are a morphological hybrid of lymphocytes and plasma cells.

Cells secrete IgM paraprotein.

205
Q

What are the clinical findings of Waldenstrom’s macroglobulinaemia?

A

Insidious onset in 60s-70s

Fatigue related to anaemia

Hyperviscosity

–> mucosal and GI bleeding, retinal vein engorgement due to engorged blood vessels and platelet dysfunction

–> neuro sx, nausea, vertigo, visual changes, stupor, coma

–> cold agglutinin disease (haemolysis), chronic demyelinating peripheral neuropathy

Hepatosplenomegaly

Purpura

No bone tenderness

No renal disease

No lytic lesions

206
Q

What are the lab findings in Waldenstrom’s macroglobulinaemia?

A

Anaemia, roleaux, RBC agglutination at rom temp

Anaemia is secondary to expansion of plasma volume (by 50-100%

Plasmacytic lymphocytes on smear

BMBx: Infiltration of plasmacytic lymphocytes

Hallmark: Monoclonal IgM spike on serum PeP in the B-globulin region

High serum viscosity (symptoms develop at 4x water viscosity, severe at 6x

Positive Coombs for complement

Cold agglutinin or cryoglobulin properties

207
Q

Why and how should you repeat protein electrophoresis if a suspected test for Waldenstrom’s macroglobulinaemia comes back with hypogammaglobulinaemia?

A

Temp must be maintained at body temp of 37c or the paraprotein may precipitate out. Sample should recollected and kept warm.

208
Q

What is the differential dx of Waldenstrom’s macroglobulinaemia?

A

MGUS –> W has monoclonal malignant cells

CLL –> Absence of CD5, different morphology, absence of lymphocytosis

Multiple myeloma –> BM morphology, IgM paraprotein, absence of bone and renal disease

209
Q

What is the treatment of Waldenstrom’s macroglobulinaemia?

A

Hyperviscosity syndrome (stupor, coma, pulm oedema) –> emergency –> plasmapheresis

Otherwise

Periodic plasmapheresis

FCR (Fludarabine, cyclophosphamide, rituximab)

210
Q

What treatment does all indolant malignant lymphoid diseases have in common?

A

Rituximab (anti-CD20 antibody)

211
Q

What is the prognosis of Waldenstrom’s macroglobulinaemia?

A

Median survival of 5 years, 10% alive at 15 years.

212
Q

A 78-year-old man presents with progressive lethargy for several months and the recent onset of bruising, epistaxis, headaches and visual deterioration. Physical examination shows him to be anaemic with scattered ecchymoses; he has reduced visual acuity bilaterally and fundoscopy reveals retinal venous distension with numerous retinal haemorrhages.

Investigations show:

The most appropriate initial therapy is:

A. melphalan and prednis(ol)one.

B. platelet transfusion.

C. plasma exchange.

D. renal dialysis.

E. intravenous gamma globulin.

A

C. This is hyperviscosity syndrome in Waldenstrom’s macroglobulinaemia.

213
Q

What are the diagnostic essentials of primary amyloidosis?

A

Congo red positive amyloid protein on tissue biopsy

Amyloid protein is kappa or lambda Ig light chain

Serum or urine (or both) positive for light chain paraprotein

214
Q

What is the pathogenesis of amyloidosis?

A

Disturbed translational or post-translational protein folding –> input of protein into tissues exceeds output –> abnormal tissue deposition –> organ dysfunction and failure

215
Q

What are the four main categories of amyloidosis?

A
  1. Primary. Ig light chain (AL Amyloidosis)
  2. Secondary. Serum protein A, produced in inflammatory conditions (AA amyloidosis)
  3. Hereditary (Transthyretin (TTR), senile amyloid (ANP) + tohers
  4. Renal Failure type (B2 microglobulin not filtered out by dialysis membranes) AB2M amyloidosis
216
Q

What are the clinical findings of amyloidosis?

A

Localised: SSx related to organ i.e. hoarseness, proptosis

Systemic:

Cardiac failure (infiltratrive/restrictive cardiomyopathy)

Nephrotic syndrome

Malabsorption and weight loss

Autonomic insufficiency

Carpal tunnel syndrome (bilateral)

Sensorimotor peripheral neuropathy

Enlarged tongue, waxy plaques on skin, periorbital contusions, cough or dyspnoea, decreased swallowing

Insidious onset, dx made late in disease

217
Q

What are the lab findings of amyloidosis?

A

Dx requires tissue biopsy that stains pink on H&E, red on Congo red, that becomes apple-green when the light is polarised.

Kappa or lambda (lambda more common) light chains in serum urine on PEP, IFE or free light chain assay in 90%, mass spectroscopy will catch the rest on tissue bx

Systemic: Blind aspiration of abdominal fat pad will reveal amyloid 66% of the time.

Ventricular wall thickening on TTE with unique speckling pattern, low QRS voltages

Renal: nephrotic range albuminuria; late: renal failure

218
Q

What is the ddx of primary amyloidosis?

A

MGUS, MM, or other malignant lymphoproliferative diseases.

12% of MGUS patients will convert to AL amylodoisis

1/5 of patients will meet crtieria for MM

5% of MM patients will have amyloid deposition of their paraprotein

219
Q

What is the treatment of primary amyloidoiss?

A

Reduce light chains to protect organs

  • melphalan and pred
  • ASCT but not generally tolerated, and can’t be done if advanced cardiac amyloidosis

Thalidomide being tested

220
Q

What is the prognosis of primary amyloidosis?

How is response and progress measured?

A

Prognostic indicators: BNP, tropT and tropI regardless of overt cardiac involvement

Poor survival, even with ASCT, median sruvival approaches 5 years

Monitored with serum free light chains.

221
Q

What is amyloid?

A

Triple-stranded fibril that forms beta pleated sheets.

Composed of amyloid protein P, and glycosaminoglycan.

222
Q

What is serum amyloid A?

A

Serum amyloid A protein (SAA) is an acute-phase reactant that is deposited in the tissues in AA amyloidosis. Involved in cholesterol transport.

223
Q

What is serum amyloid P?

A

Basic component of amyloid regardless of amyloid disease type.

224
Q

The following proteins are all components of amyloid deposits. Which is universally present?

A. Transthyretin.

B. Serum amyloid P (SAP).

C. β2-microglobulin.

D. Immunoglobulin light chain.

E. Serum amyloid A (SAA).

A

B

225
Q

What are the diagnostic essentials of heparin induced thrombocytopaenia?

A

Thrombocytopaenia within 5-10 days of espoxure to heparin

Drop in baseline PLT count of 50% or more

Thrombosis in 50% of cases, bleeding is uncommon

226
Q

What is the pathogenesis of HITS?

A

Heparin > LMWH –> formation of antibodies to heparin-platelet factor 4 complexes –> antibodies bind platelets –> activates them –> thrombocytopaenia, pro-thrombotic state

227
Q

What are the clinical findings in HITS?

A

Asymptomatic

Thrombosis (venous or arterial) may be detected in up 50% of patients up to 30 days post dx

228
Q

What are the lab findings in HITS?

A

New onset thrombocytopaenia within 5-10 days of exposure to heparin (rapid onset HIT is not common)

Drop of PLT count to 50% or more of baseline

Positive 4T score

Dx confirmed through positive PF4-heparin antibody enzyme-linked immunosorbent assay (ELISA) or functional assay (serotonin release assay) or both

Magnitute of positive ELISA result correlates with clinical probability of HIT.

229
Q

What does the serotonin release assay measure?

A

Measure serotonin release - a marker of platelet activation

If release is high, the test is positive and the platelets are activated. Used in HIT.

230
Q

What is the treatment of heparin-induced thrombocytopaenia?

A

STOP all forms of heparin.

Doppler lower limbs

Start alternative anticoagulant –> usually a direct thrombin inhibitor (argatroban or lepirudin). Continue until PLT 100 then start Warfarin.

If using argatroban, cease infusion prior to measuring INR 2 hours early so you know the anticoagulant effect is from Warfarin alone

Warfarin contraindicated initially as may transiently worsen hypercoagulability.

Continue Warfarin for 30 days due to persisten risk of thrombosis even if normal platelet count.

If HITS plus thrombosis, Warfarin for 3-6 months

Avoid subsequent exposure to Heparin, if absolutely necessary, measure PF4-antibdoies which should be negative (at about 100 days) and limit exposure to heparin.

231
Q

What is Trousseau’s syndrome?

A

Sign found in certain cancers that is associated with venous thrombosis and hypercoagulability. It is characterized by successive crops of tender nodules in affected veins.

232
Q

What are the 4Ts in the diagnosis of HITs?

Predictive value?

A
  1. Thrombocytopaenia - 2 points if the count falls >50%, no points if it’s less than <30%
  2. Timing of platelet count fall. 2 points if 5-10 days or if PLT count falls <1 day with heparin exposure within 30 days.
  3. Thrombosis. 2 points if new thrombosis, skin necrosis, or acute systemic reaction to unfractionated heparin bolus.
  4. Other causes of Thrombocytopaenia. 2 points if none. Zero points if you can find another cause.

Very good at excluding cases of HITs (NPV of almost 1). Low probability means you can exclude, intermediate and high require further investigation.

233
Q

What is the d-dimer?

A

Fibrin degradation product present in blood after a clot is degraded by fibrinolysis - contains two crosslinked fragments of fibrin protein

234
Q

What are the diagnostic essentials of DIC?

A

Prolonged APTT and PT

Decreased fibrinogen

Thrombocytopaenia

235
Q

What is the mechanism of DIC?

A

Uncontrolled local or systemic activation of coagulation —> depletion of coagulation factors and fibrinogen, activation and consumption of platelets leading to thrombocytopenia

236
Q

What are the causes of DIC?

A

Sepsis - coagulation activated by LPS

Cancer

Trauma

Burns

Pregnancy associated (tissue factor)

Aortic aneurysm & cavernous haemangiomas

Snakebited due to exogenous toxins

237
Q

What are the signs and symptoms of DIC?

A

BLEEDING esp at IVCs or incisions

May be widespread (purport fulminates)

Malignancy related - thrombosis (trousseau syndrome)

238
Q

What are the lab findings in DIC?

A

Acute and progressive prolongation of coats and thrombocytopenia

Early: PLT and fibrinogen may remain normal

Progressive thrombocytopenia

Elevated d-dimer due to activation of coagulation and diffuse cross-linking of fibrin

Schistocytes on blood smear due to shearing through microvasculature

HELLP of pregnancy: haemolysis, deranged LFTs, thrombocytopenia, renal dysfunction with gross haemoglobinuria and pigment nephropathy

Malignancy-related: normal PLT counts and coags

239
Q

What is the treatment of DIC?

A

Treat underlying cause

Monitor PT aPTT fibrinogen and PLT count 6 hourly

Give PLTs and aim count >20, or >50 if bleeding

Cryo to replace fibrinogen (>8-10)

FFP to get PT and aPTT down

PRBCs to get Hb >80

Heparin if persistent bleeding to stop thrombin generation and thus consumption of coal proteins and platelets. Contraindicated if PLT <50, bleeding, placental abruption or any need for surgery

HELLP —> evacuate uterus (deliver baby, remove retained placenta or fetal fragments)

Trousseau syndrome —> treat malignancy and give heparin to treat thrombosis (Warfarin doesn’t work)

APL associated DIC —> immediate (within 24h) chemotherapy plus blood products as required.

240
Q

What is HELLP?

How do you treat it?

A

HELLP of pregnancy: haemolysis, deranged LFTs, thrombocytopenia, renal dysfunction with gross haemoglobinuria and pigment nephropathy

leads to DIC

Evacuate the uterus.

241
Q

What are the risk factors for acquired antibodies to Factor VIII?

A

Elderly

Malignancy

CTD

Postpartum or postsurgical

242
Q

What are the findings in acquired antibodies to Factor VIII?

A

Soft-tissue ecchymoses, haematomas, mucosal bleeding (as opposed to haemoarthrosis in congenital haemophilia A)

243
Q

What are the lab findings if you have acquired antibodies to factor VIII?

How do you treat it?

A
  • prolonged APTT that does not correct on mixing
  • low factor VIII activity
  • Bethesda assay will give you titre

Treatment

Low titre - give high doses of factor VIII

High titre - prothromboninex or recombinant activated factor VII

  • plus steroids or cyclophosphamide

Refractory: IVIG, ritux or plasmapheresis

244
Q

What is the red cell distribution width?

A

Measure of the variation of red blood cell (RBC) volume that is reported as

part of a standard complete blood count. Usually red blood cells are a standard size of about 6-8 μm in diameter. Certain disorders, however, cause a significant variation in cell size. Higher RDW values indicate greater variation in size.

245
Q

What is the diagnostic criteria for hereditary haemorrhagic telangectasia?

A

Four criteria, need 3/4 for definite, 2/4 for posible.

  1. Spontaneous recurrent epistaxis
  2. Multiple telangiectasias in typical locations
  3. Proven visceral AVM (lung, liver, brain, spine)
  4. First-degree family member with HHT
246
Q

What are the classical manifestations of hereditary haemorrhagic telangectasia?

A

Spontaneous, recurrent epistaxis - 90%

Skin telangiectases - 75%

Hepatic or pulmonary involvement (arteriovenous malformations [AVMs]) - 30%

Gastrointestinal (GI) bleeding - 15%[31]

CNS lesions inc headache

247
Q

What does GPIIb-IIIa do?

A

Expressed on activated platelets

helps platelets adhere to damaged endothelium and to stick to one another

248
Q

Intrinsic - XII XI

A

IX

249
Q

What clinical phenotype and clotting study would you see in FXII (factor 12) deficiency? Prolonged APTT

A

but no clinical change on bleeding time

250
Q

Has it normalised? (disclaimer: numbers don’t correlate to actual reference ranges; I made it up in my head) NO! The mixed 1 : 1 is in the normal range

A

but it has not normalised. Need to get within 4-5 seconds of the control (in this example up to 28s) to have ‘normalised’; 32 shows there’s still some inhibitor at work

251
Q

Name 2 causes of prolonged PT that correct with mixing studies PT = extrinsic pathway

A

ie Factor 7!

252
Q

Mod

A

1-5% - minor trauma gives bleed

253
Q

T/F? Getting haemophilia from a de novo mutation is rare - you nearly always inherit it False; 1/3rd of pts don’t have a family history

A

ie haemophilia mutation came on spontaneously

254
Q

Tranexamic acid is particularly useful for stabilising bleeding / clots in which area of the body? GI tract & mouth

A

good for dental procedures - use as mouthwasy or oral tablets

255
Q

What is the Bethesda assay as it relates haemophilia? Both qualifies (presence / absence) and quantifies (the higher the titre

A

the stronger) inhibitors against (recombinant) factor

256
Q

What’s the most common bleeding disorder in the community? vWF

A

1%

257
Q

In general

A

all pts with mild Type 1 vWD and pts with mild haemophilia A should receive what treatment as first-line therapy (prophylaxis/treatment) of low risk ops / slight bleeds? DDAVP

258
Q

What causes HITs? Antibody-mediated platelet activation

A

antibodies bind to platelet factor 4 (PF4)

259
Q
  • thrombocytopenia or >50% fall in plts from baseline
A

AND

260
Q
  • start anticoagulant - danaparoid cover (or direct thrombin inhibitors
A

eg lepirudin or argatroban or fondaparinux) and start warfarin for 3 months (as HITs is a procoagulant state)

261
Q

What’s the management of an INR 2.5 - 4.5 but no bleeding? Withhold or reduce next dose of warfarin

A

and restart when INR lower

262
Q

What’s your warfarin plan for someone with low risk of clots getting a low bleeding risk procedure (eg cataracts

A

dental work)? Keep on with the warfarin

263
Q

What’s the difference between forward grouping & reverse grouping for blood type sorting? Forward - mix pt RCs with anti-A

A

anti-B and anti-Rh D antibodies. Checks for antigens on pt’s RCs. Agglutination = positive.

264
Q

With regards to preparing blood products for transfusion

A

why do we do leucodepletion? Why do we do irradiation? Leucodepletion - to reduce febrile non-haemolytic transfusion reaction due to HLA sensitisation or release. “CMV safe”.

265
Q

Name 2 acquired thrombophilic conditions & two hereditary thrombophilias? Acquired: antiphospholipid syndrome

A

cancer

266
Q

Why is ‘lupus anticoagulant’ a misnomer? Because it’s not an anticoagulant

A

it’s a prothrombotic agent

267
Q

What is your management for a lady with APLS who is now pregnant

A

who has had prior clots but no pregnancy complications? Therapeutic anticoagulation

268
Q

T/F? Rivaroxaban can be dialysed off False; highly protein bound

A

can’t HDx off

269
Q

T/F? Dabigatran inhibits free thrombin

A

but not thrombin that is bound to clots False; inhibits both free & clot-bound thrombin

270
Q

T/F? Dabigatran primarily excreted by liver False

A

80% renally excreted

271
Q

Name two PBS indications for rivoraxaban VTE prophylaxis in knee & hip surgery (dabigatran

A

apixiban too)

272
Q

What’s the most common mode of inheritance of vWD? AD (Type 1

A

which is vast majority of cases)

273
Q

Less common: anaemia of chronic disease

A

hyperthyroidism

274
Q

There are two forms of non-heme iron: ferric and ferrous. Which type do we absorb best? We absorb ferrous (Fe2+) best - 2

A

like ‘me too!’. 3 is not good at being absorbed and our body actively changes Fe3+ to Fe2+ (via duodenal cytochrome B).

275
Q

Which one is ferric / ferrous iron? Fe2

A

Fe3? Ferric Fe3 - ric is ick! We don’t like it

276
Q

Ferrous Fe2 - ‘ous’ is delicious

A

we like this best

277
Q

How long should someone get oral iron replacement for iron def anaemia? 6 months to replace stores. Monitor Hb monthly (should increase 20-30 / month)

A

and reticulocytes should increase after 1-2 weeks (in response to making mor eiron)

278
Q

In what conditions is hypoxia inducible factor (HIF) released? Released in iron deficiency / hypoxia. Up regulates DyctB and DMT1

A

increases iron absorbtion

279
Q

Complete this sentence: dietary B12 is bound to R proteins

A

liberation depends on xxxx pancreatic enzymes. Dietary B12 will remain bound to R protein in pts with pancreatic insufficiency (which is therefore a cause of B12 deficiency)

280
Q

Where is B12 absorbed? Terminal ileum

A

via cubilin receptor

281
Q

T/F? Both B12 & folate deficiencies can cause neurologic manifestations No

A

only B12 deficiency can cause neuro issue (subacute degeneration of spinal cord); folate doesn’t affect neurologic system in such a way

282
Q

What’s the most sensitive test for haemolysis? (low) Haptoglobin - if normal

A

patient very unlikely to have significant degree of haemolysis.

283
Q

Urinary haemosiderin is low / normal / high in a) intravascular haemolysis

A

b) extravascular haemolysis a) intravascular- positive urinary haemosiderin

284
Q

Mechanical haemolytic anaemia (prosthetic valve

A

severe valve disease)

285
Q

What’s the pathophysiology of PNH? Rare disease (1 in 1 million)

A

acquired

286
Q

Cold at < 37 degrees (eg. 34

A

4 degrees)

287
Q

T/F? Alpha thalassemia can be diagnosed on electrophoresis? False

A

can be diagnosed only on DNA studies (electrophoresis for Beta-thal)

288
Q

To make gross generalisations

A

what blood disorder to you see in SE Asia? Mediterranean? Africa? SE Asia - alpha thal

289
Q

T/F? HbH is low affinity for oxygen False

A

high affinity for oxygen

290
Q

a+ is deletion / inactivation of one allele of a gene (but residual function of the other allele

A

so the gene itself is still -partially - functional)

291
Q

T/F? Ferritin levels in thalassaemia correlate well with iron burden in heart & liver False

A

doesn’t correlate well. Need annual cardiac MRI to check iron burden on organs.

292
Q

What’s the role of hydroxyurea in sickle cell disease? For all sickle cell disease (also used in PRV & ET)

A

helps increase HbF levels & prevent crisis by increased Hb O2 binding capacity

293
Q

Auer rods

A

hypergranular promyelocytes = ? APML

294
Q

Idiopathic abdominal vein thrombosis (eg Budd Chiari) is an indication to test for what mutation?

A

JAK2 mutation testing to look for essential thrombocytosis

295
Q

What’s the key difference between myeloproliferative disorders & myelodysplastic disorders? As it sounds - ‘proliferative’ - cells are fine

A

there’s just too many of them

296
Q

Dysplastic’ - deformed cells

A

and not enough of them

297
Q

What’s the treatment for low grade NHL? Chlorambcil

A

fludarabine

298
Q

With regards to MGUS

A

what’s the biggest predictor of progression to MM? Size of the myeloma protein

299
Q

Post allograft

A

when does engraftment usually occur? Day 15-30

300
Q

Which body part most common involved in acute GVHD?

A

Skin 90%. Then liver & gut. These three main sites are involved in GVHD occuring before Day 100 of allograft.

301
Q

T/F? GVHD involving liver p/w hepatitic LFT derangment False

A

usually p/w cholestatic LFTs. DDx: veno-occlusive disease

302
Q

Hepcidin is involved in control of iron absorption. Which of the following conditions would be a/w high hepcidin. A) Anaemia. B) Iron deficiency. C) Thalassaemia. D) Sideroblastic anaemia. E) Haemochromatosis. F) Chronic inflammation. Both Anaemia

A

Chronic inflammation.

303
Q

The JAK2 mutation is in what % of PRV

A

and what % of ET? PRV - 95%

304
Q

Are the following cytogenetics favourable are unfavourable? inv(16)

A

t(8

305
Q

Are the following cytogenetics favourable are unfavourable? inv(16); t(8;21); t(15;17)

A

Favourable

306
Q

In ALL, which of the following are favourable? Which are unfavourable? BCR-ABL, MLL rearrangement

A

Both are unfavourable; they’ll do poorly. Need aggressive post-remission therapies

307
Q

What’s imatinib, and how is it used?

A

It’s molecular targeted therapy at tyrosine kinase, added on to chemoTx, and has drastically improved prognosis of CML

308
Q

The cytogenetic marker, ‘del 5q’ is found in what condition?

A

Subtype of MDS. This type is responsive to lenalidomide.

309
Q

In which part of the GIT is iron absorbed?

A

Duodenal enterocytes

310
Q

Where in the body is transferrin synthesised?

A

In liverr

311
Q

There are two types of transferrin receptors (types 1 & 2). Where are they located, and which is implicated in HH?

A

Type 1 - major binding port for iron, found on most cells. Binds transferrin 30x stronger than TfR2.

312
Q

What is ferritin?

A

It’s a storage facility for iron. Ferritin consists of an apoprotein shell with light & heavy chain subunits, which surround a core of iron atoms.

313
Q

How do ferritin levels help us work out how much iron is in the body?

A

Cells excrete the same amount of ferritin storage units into the blood stream as they currently have inside them, and so checking the ferritin levels in the blood gives us an indication of how many ferritin ‘storage units’ there are within the cells, and from there, we can guess what amount of iron is present in the cell

314
Q

What’s the main driver of ferritin production?

A

Iron in cells.

315
Q

What’s the link between hepcidin and ferroportin?

A

Hepcidin causes internalisation of ferroportin, and so iron has no way of leaving the cell (it’s only exit is via ferroportin). When body has low iron in storage, hepcidin expression is reduced (to allow ferroportin to stay on cell membrane & get iron out of cell into blood); and vice versa

316
Q

Which type of HD carries the best prognosis?

A

Lymphocyte predominant

317
Q

T/F? With regards to Hodgkins Disease, the presence of B symptoms implies a better prognosis

A

False - worse prognosis

318
Q

Name 2 poor prognostic factors for HD?

A

As per NEJM 1998

319
Q

Name an alkylating agent

A

Cyclophosphamide - causes cross-linking of DNA. S/Es: haemorrhagic cystitis, myelosuppression, transitional cell carcinoma

320
Q

Name a S/E of bleomycin

A

Lung fibrosis

321
Q

Name a S/E of doxorubicin

A

Cardiomyopathy

322
Q

Name 2 anti-metabolite drugs

A

MTX

323
Q

Name a chemoTx agent that causes peripheral neuropathy

A

Vincristine

324
Q

Name 2 modalities which may help improve metastatic bone pain

A

NSAIDs

325
Q

Name cytogenetic abnormalities for: a) CML, b) APML, c) Burkitt’s, d) Mantle cell lymphoma

A

a) CML - t(9:22) - Philadelphia chromsome; BCR-AbL

326
Q

What’s the first-line treatment for pyoderma gangrenosum?

A

Oral steroids; if not resolving, try immunosuppressive therapy (ciclosporin, infliximab)

327
Q

Name 2 causes of warm AIHA

A

AID, eg SLE

328
Q

Name 2 causes of cold AIHA

A

Neoplasia, eg lymphoma

329
Q

How do you treat warm AIHA?

A

Steroids

330
Q

How do you diagnosis hereditary haemorrhagic telangiectasia?

A

Aka Osler-Weber-Rendu syndrome, is an AD condition characterised by multiple telangiectasia over skin / MMs. If you meet 2/4 of following = ‘possible’ diagnosis, if you meet 3/4 or 4/4 = ‘definite’ diagnosis. 1) Recurrent, spont. epistaxis. 2) Telangiectases (lips, mouth, fingers, nose). 3) AVMs in GIT, hepatic, cerebral, spinal, lungs. 4) FHx (1st degree relative) with HHT

331
Q

In pts with sickle cell anaemia, what viral infection can precipitate aplastic anaemia?

A

Parvovirus

332
Q

Is paroxysmal nocturnal haemoglobinuria inherited or acquired?

A

Acquired disorder leading to haemolysis of haematologic cells, thought to be due to increased sensitivity of cell membranes to complement due to lack of glycoprotein glycosyl-phosphatidylinositol

333
Q

T/F? PNH makes pts more exposed to VTE

A

True

334
Q

What’s the gold standard investigation for PNH?

A

CD59 & CD55 on flow cytometry

335
Q

Name 3 features of PNH

A
  • Haemolytic anaemia
336
Q

Is hereditary spherocytosis AD or AR inheritance?

A

AD

337
Q

What’s the underlying cause of hereditary spherocytosis? ie) what bit is abnormal, and how?

A

RC cytoskeleton is deformed; no longer biconcave disc shape but rather sphere-shaped. This reduces RC survival –> destroyed by spleen

338
Q

Osmotic fragility test diagnoses what?

A

Hereditary spherocytosis

339
Q

Hereditary spherocytosis can be diagnosed by what?

A

Osmotic fragility test

340
Q

What’s the inheritance pattern of G6PD deficiency?

A

Male (X-linked recessive)

341
Q

What is seen on blood films in G6PD deficiency?

A

Heinz bodies

342
Q

What is seen on blood films in hereditary spherocytosis?

A

Spherocytes (round, lack of central pallor)

343
Q

What’s the diagnostic test for G6PD deficiency?

A

Measure enzyme activity of G6PD

344
Q

T/F? Haemochromatosis is a/w polycythemia

A

False. Not a/w polycythemia. Usually blood tests show raised ferritin & iron, a/w transferrin saturation > 60% and a low total iron binding capacity

345
Q

Name 2 causes of polycythemia

A

Dehydration

346
Q

Having the Philadelphia chromosome (t 9:22) is a good prognostic marker in one disease, and a bad prognostic marker in another. Can you name them?

A

Philadelphia chromosome GOOD in CML, but BAD in AML & ALL

347
Q

Burkitt’s lymphoma is a/w what gene translocation?

A

c-myc, usually t: 8:14

348
Q

The c-myc gene translocation (t 8:14) is a/w what haematologic malignancy?

A

Burkitt’s lymphoma

349
Q

Unfractionated heparin forms a complex to inhibit what factors? How about LMWH?

A

UFH - activates antithrombin III. Inhibits thrombin, factors Xa, IXa, XIa, XIIa

350
Q

How is unfractionated heparin monitored (what blood test)? How about LMWH?

A

UFH - APTT

351
Q

HITs is caused by what?

A

Immune mediated-antibodies, which activate platelets. This results in low platelet count (>50% reduction), (however HITs paradoxically is a prothrombotic condition).

352
Q

What alternative treatment could you use in someone who develops HITs?

A

Lepirudin

353
Q

What is imatinib? When is it used?

A

Imatinib = tyrosine kinase inhibitor. Used in CML. Specifically inhibits the tyrosine kinase associated with the BCR-ABL defect.

354
Q

How many pts (%) with CML have Philadelphia chromosome? (t 9,22)

A

95%

355
Q

What’s the first line management of chronic CML?

A

Imatinib is FIRST LINE TREATMENT

356
Q

What are the findings on immunophenotyping in CLL?

A

B-cells (CD 19 positive)

357
Q

A patient has excessive bleeding post a dental procedure. Found to also have a petechial rash, slightly elevated APTT and reduced FVIII activity. ? Dx

A

vWD

358
Q

What’s the most common inherited bleeding disorder?

A

vWD

359
Q

Is vWD inherited AD, AR or X-linked?

A

AD

360
Q

What are the 3 types of vWD?

A

1) Partial reduction in vWF (80% of pts, AD)

361
Q

What’s the link between DDAVP and vWF?

A

Desmopressin (DDAVP) raises level of vWF by inducing release of vWF from Weibel-Palade bodies in endothelial cells

362
Q

Which cells in the stomach secrete intrinsic factor?

A

Parietal cells

363
Q

Where is Vitamin B12 absorbed?

A

Terminal ileum

364
Q

Name 2 causes of Vit B12 deficiency

A

Pernicious anaemia

365
Q

What’s the single most important test in determining prognosis in AML?

A

Cytogenetics (although gene-expression profiling, WCC at diagnosis, immunophenotyping, LDH are also used to help prognosticate)

366
Q

Patients with deficiency in what part of the clotting cascade may be resistant to heparin treatment?

A

Antithrombin III deficiency - heparin works by enhancing its action.

367
Q

What’s first line management of ITP?

A

High dose prednisolone

368
Q

What would you see on BMAT for someone with ITP?

A

Increased megakaryocytes

369
Q

In ITP, immune complexes are directed against what?

A

Glycoprotein IIb-IIIa complex

370
Q

What’s the Mx of ITP?

A

Oral PNL (80% of pts respond)

371
Q

What’s the main cause of hypercalcemia in MM?

A

Due primarily to increased osteoclastic bone resorption caused by local cytokines (IL1, TNF) released by myeloma cells

372
Q

Which of the following have been shown to increase risk of DVT? Tamoxifen, raloxifene, HRT, OCP

A

All of them

373
Q

Name the two types of immunoglobulins most commonly produced in myeloma

A

IgG and IgA

374
Q

What is Waldenstrom’s macroglobulinemia?

A

Uncommon, seen in old men. Lymphoplasmacytoid malignancy, characterised by secretion of monoclonal IgM paraprotein. Features: weight loss, hyperviscosity / DVTs, hepatosplenomegaly, LNs, cryoglobulinemia eg Raynaud’s

375
Q

T/F? Thymomas a/w red cell aplasia

A

True

376
Q

What’s the initial treatment for CLL?

A

Fludarabine

377
Q

What’s the relationship between TTP & ADAMTS13?

A

TTP due to abnormally large and sticky multimers of vWF which cause plts to clump within vessels. This is due to a deficiency of ADAMTS13, which normally breaks down large multimers of vWF.

378
Q

Tear-drop poikilocytes seen on blood film. Diagnosis?

A

Myelofibrosis.

379
Q

What is the condition in which you see: target cells, Howell-Jolly bodies, Pappenheimer bodies, siderotic granules, acanthocytes? (one condition!)

A

Hyposplenism, eg post splenectomy

380
Q

What is the condition in which you see: target cells, ‘pencil’ poikilocytes?

A

Iron defiency

381
Q

What is the condition in which you see schistocytes?

A

Intravascular haemolysis

382
Q

The combination of prolonged APTT & low platelets is seen in what prothrombotic condition?

A

Antiphospholipid syndrome

383
Q

Is hereditary haemorrhagic telangiectasia AD or AR or X- linked inheritance?

A

AD

384
Q

T/F? Trimethoprim may cause pancytopenia

A

True

385
Q

Name 3 drugs that can cause pancytopenia

A

Cytotoxic

386
Q

What anticoag should be used in a pregnant lady with a DVT?

A

Clexane. Less bleeding risk than heparin.

387
Q

What’s the management of thrombotic pnacytopenia?

A

Hydroxyurea

388
Q

What’s the relationship between activated protein C resistance and Factor V Leiden mutation?

A

APC resistance is due to mutated FVL.

389
Q

What’s the risk of having VTE in someone heterozygote for FVL? What about someone homozygous for factor V Leiden?

A

Heterozygote - 5x increased risk of VTE

390
Q

What do you do to prevent haemorrhagic cystitis in someone getting cyclophosphamide?

A

Mesna

391
Q

How dose mesna prevent haemorrhagic cystitis in pts getting cyclophosphamide?

A

It binds to the urotoxic metabolites produced by cyclophosphamide, which reduces then the incidence of haemorrhagic cystitis

392
Q

T/F? Cyclophosphamide is an alkylating agent?

A

True. Works by cross-linking DNA.

393
Q

What’s the treatment of choice for TTP?

A

Plasma exchange first-line.

394
Q

Is haemochromatosis inheritance AR or AD?

A

AR

395
Q

In the general population, what is the most useful marker for screening for haemochromatosis?

A

Transferrin saturation

396
Q

A liver biopsy is positive for Perl’s stain. Diagnosis?

A

Perl’s stain = iron; pt has haemochromatosis

397
Q

What would you expect regarding the following parameters in someone with HH? Transferrin saturation, ferritin, and total iron binding capacity? (low, normal, high)

A

Transferrin saturation - high

398
Q

With regards to HH, monitoring of what parameter is best to assess adequacy of venesection?

A

Ferritin

399
Q

What % of pts with MGUS go on to get MM in 5 yrs?

A

10% at 5 yrs;

400
Q

T/F? B2-microglobulin is a useful marker of prognosis in MM?

A

True; a raised level implies poor prognosis

401
Q

Reed-Sternberg cells = what diagnosis?

A

Hodgkin’s lymphoma

402
Q

T/F? Hodgkin’s lymphoma is commonest in the young (20s) and older ppl (60s)

A

True

403
Q

What’s the Ann-Arbor staging of Hodgkin’s lymphoma?

A

Ann-Arbor staging of Hodgkin’s lymphoma

404
Q

What’s the most common type of Hodgkin’s lymphoma?

A

Nodular sclerosing

405
Q

Name 3 poor prognostic factors in CLL

A

Male

406
Q

5-15% of ppl with polylcythemia rubra vera progress to what?

A

Myelofibrosis or AML

407
Q

What’s the management of polycythemia rubra vera?

A

Venesection - first line treatment

408
Q

What’s the best test to screen the gen pop for HH? How about an asymptomatic first degree relative of someone with known HH?

A

Gen pop = use transferrin saturation (better than ferritin level)

409
Q

Would amiodarone enhance or decrease warfarin’s function? (ie increase or decrease INR?)

A

Increase / enhances warfarin, ie expect an increased INR with concommitant use of warfarin

410
Q

Do people with rheum arthritis get thrombocytosis or thrombocytopenia?

A

Tend towards thrombocytosis

411
Q

Which haematological malignancy is a/w gingival hyperlasia?

A

AML

412
Q

Anti-thymocyte globulin (ATG) and anti-lymphocyte globulin (ALG) is used in what condition?

A

Aplastic anaemia

413
Q

What’s the most common bleeding disorder? What’s the most common clotting disorder?

A

Bleeding - vWF

414
Q

T/F? Plasma exchange is first line therapy for ITP

A

False. First line is a trial of oral PNL; 80% of pts respond. If not responding, do splenectomy, +/- IVIg +/- immunosuppressive drugs

415
Q

What diseases are thymomas associated with?

A

Thymomas are the most common tumour of the anterior mediastinum

Associated with

myasthenia gravis (30-40% of patients with thymoma)

red cell aplasia

dermatomyositis

also : SLE, SIADH

416
Q

What malignancies are ABV, HTLV-1 and HIV-1 associated with?

A

EBV: Hodgkin’s and Burkitt’s lymphoma, nasopharyngeal carcinoma

HTLV-1: Adult T-cell leukaemia/lymphoma

HIV-1: High-grade B-cell lymphoma

417
Q

What are the high and low risk chemotherapeutic agents with regard to infertility?

A

High - ALKYLATORS

  • cyclophosphamide
  • chlorambucil
  • procarbazine
  • melphalan

Low

Antimetabolites

Vincristine

Bleomycin